SlideShare a Scribd company logo
Course Overview
3 of 21
Welcome and Introductions
• Course Director
• Faculty
• Course Coordinator
Course Overview
4 of 21
Welcome and Introductions
We would like you to introduce yourselves now:
Who are you?
What is your professional background?
What is your experience with trauma?
What do you hope to gain from this course?
Course Overview
5 of 21
Safe Learning Environment
• Interactive, unfolding case discussions, stimulus questions
• Participation required
• Respect for each other
• Try answering the questions – it’s ok if you get it wrong
Course Overview
6 of 21
M
43 year old male, driver MVC,
involved in head on collision
with truck
I
Right sided bruising and
abrasions on Chest, deformed
right leg,
S
Non responsive, RR 30, HR 130,
BP 80/60
T
On spine board with C spine
collar
Discussion Question:
What are your concerns with this
patient?
Course Overview
7 of 21
The ATLS course provides one
acceptable method for the safe,
immediate management of
trauma patients.
Course Overview
8 of 21
Program Goals
ATLS course provides participants with a safe and reliable method to:
1. Assess a patient’s condition rapidly and accurately.
2. Resuscitate and stabilize patients according to priority.
3. Determine whether a patient’s needs exceed a facility’s resources and/or provider’s
capabilities.
4. Arrange transfer when indicated
A
?⃝
→ Oxygenation flow to cells
⑦ →
eoagulophaty
Course Overview
9 of 21
Course Objectives
Upon completion of the ATLS student course, you will be able to:
1. Demonstrate the concepts and principles of the primary and secondary
patient assessments.
2. Establish management priorities in the initial management of a trauma
patient.
3. Initiate the primary and secondary management of a simulated trauma
patient in a timely manner.
4. In a given trauma simulation, demonstrate the skills that are often required
in the initial assessment and treatment of patients with multiple injuries.
Airway → 5 ppnypbab → baca
buta
TABCDE
Course Overview
10 of 21
The Need
• 5.8 million people die every year
from unintentional injury and
violence -- more than nine people
every minute.
• Injury accounts for 18% of the
world’s burden of disease.
• Motor vehicle crashes alone cause
more than 1 million deaths annually
and 20 to 50 million significant
injuries.
Self-inflicted
violence
16%
Interpersonal
violence
10%
Other
17%
Road traffic
injuries
25%
Fires 5%
Poisoning 6%
Falls 6%
War
6%
Drowning
9%
Course Overview
11 of 21
ATLS provides a
common language
Course Overview
12 of 21
The Beginning
Course Overview
13 of 21
“When I can provide better
care in the field with
limited resources than what
my children and I received
at the primary care facility,
there is something wrong
with the system, and the
system has to be changed.”
James Styner, MD, FACS
1977
Course Overview
14 of 21
Trimodal to Bimodal Distribution
L % Jam → 5 days
death
1 JM → besok
71 Jan → hari in
Course Overview
15 of 21
ATLS Concept
• Follow ABCDE approach to evaluation and treatment.
• Treat the greatest threat to life first.
• Recognize the definitive diagnosis is not immediately important.
• Understand that time is of the essence.
• Do no further harm.
✗
ray thorax e
pelvis
Course Overview
16 of 21
ATLS Concept
Airway with restriction of cervical spine motion
Breathing and ventilation
Circulation with hemorrhage control
Disability: Neurological status
Exposure / Environmental control
Bleeding
4) I II IT IV
≤750 ≤ 1500 ≤2000712000
Course Overview
17 of 21
Initial Assessment and Management
Course Overview
18 of 21
ATLS Educational Format
• Introductory lecture
• Interactive group discussions
• Interactive skill sessions
• Simulated patient scenarios
• Written examinations
• mATLS online learning modules
• MyATLS mobile app
Course Overview
19 of 21
International ATLS Program
• 83 countries
• 3,380 courses
• 68,000 students
• MyATLS mobile app 181 countries,
216,000 downloads
Course Overview
20 of 21
Impact of ATLS Program
• Documented improvement in the care of injured patients after
implementation of program
• Organized trauma care resulting in reduced injury mortality
• Retention of organizational and procedural skills
Course Overview
21 of 21
Any Questions?
ABC → fangs, wana
Jaknnya Coagulopathy
→ ttoas of
AS/dos's
Death hepotrensi
↳ Korans Imprint → 6 man't
↓ Man blasts
Kamata Kling
Course Overview
22 of 21
Summary
• ABCDE approach to trauma care
• Do no further harm
• Treat the greatest threat to life first
• One safe way
• A common language
Course Overview
23 of 21
Video 1
Course Overview
24 of 21
Course Overview
25 of 21
1 Initial Assessment and Management
The primary survey (ABCD) is the cornerstone of the initial
assessment of the trauma patient. Repeat the primary survey
frequently to identify any deterioration in the patient’s
status that indicates the need for additional intervention.
Course Overview
26 of 21
Objectives
By the end of this interactive discussion, you will be able to:
1. Explain the importance of preparation prior to trauma patient arrival.
2. Evaluate the mechanism of injury to determine the patient’s potential
injuries.
3. Identify the correct sequence of priorities for the assessment of a multiply
injured patient.
4. Apply the principles of the primary and secondary surveys to the assessment
of a multiply injured patient.
5. Discuss the importance of reevaluating a patient who is not responding
appropriately to initial resuscitation and management.
6. Recognize patients who require transfer to another facility for definitive
management.
Course Overview
27 of 21
1
Case Scenario
18-year-old male , unrestrained
driver in MVC vs. tree
None reported
Vitals not reported
Prolonged extrication; transported
to ED by ambulance; O2 by mask;
fluids via single IV; spinal motion
restricted on long spine board
Initial Assessment and Management
M
I
S
T
Course Overview
28 of 21
1
Discussion Questions:
1. How would you prepare for the arrival
of this patient?
2. What other information would be
helpful to know in order to prepare?
3. From the history, what are the
potential injuries this patient may have
suffered?
Initial Assessment and Management
Case Details
M
18-year-old male ,
unrestrained driver in MVC
vs. tree
None reported
Vitals not reported
Prolonged extrication;
transported to ED by
ambulance; O2 by mask;
fluids via single IV; spinal
motion restricted on long
spine board
I
S
T
Course Overview
29 of 21
1
Case Scenario Progression
• EMS report: patient is
lethargic, mumbling
unintelligibly
• Patient has facial injuries
• Vital signs: HR 120; BP 90/40;
RR 24, O2 sat 89%, temp 36°C
Initial Assessment and Management
Course Overview
30 of 21
1
Discussion Questions:
1. Based on this information, what
interventions can be done in the
prehospital setting?
2. Which patients should be immediately
transported to the trauma center
based on their field presentation?
Initial Assessment and Management
Case Details
• EMS report: patient
is lethargic,
mumbling
unintelligibly
• Patient has facial
injuries
• Vital signs: HR 120;
BP 90/40; RR 24, O2
sat 89%, temp 36°C
Course Overview
31 of 21
1
Case Scenario Progression
• Patient arrives at hospital
• Vital signs: HR 120; BP 90/palp;
RR 20; O2 sat 82%, temp 35.5°C.
Initial Assessment and Management
Course Overview
32 of 21
1
Discussion Questions:
1. What are your clinical concerns?
2. What are your management priorities?
Initial Assessment and Management
Case Details
• Patient arrives at
hospital
• Vital signs: HR 120;
BP 90/palp; RR 20;
O2 sat 82%, temp
35.5°C.
Course Overview
33 of 21
1
Case Scenario Progression
Primary survey reveals:
A: Obvious facial trauma and mumbling incoherently.
B: Decreased breath sounds, L chest; no visible neck veins
C: Minimal bleeding; open L femur fracture; L chest bruising;
possible pelvic fracture
D: Localizes to pain with upper extremities; moans to painful
stimuli; does not open eyes
Initial Assessment and Management
tmpt mpnanpurg - thorax
¢ Caution abdomm
_
retroperitoneal
darah
(Prius
hulas Panjang
Course Overview
34 of 21
1 Initial Assessment and Management
Case Details
A: Obvious facial trauma and
mumbling incoherently.
B: Decreased breath sounds, L
chest; no visible neck veins
C: Minimal bleeding; open L
femur fracture; L chest
bruising; possible pelvic
fracture
D: Localizes to pain with upper
extremities; moans to
painful stimuli; does not
open eyes
Discussion Questions:
1. What are your clinical concerns?
2. What are your management priorities?
-
Course Overview
35 of 21
1
Case Scenario Progression
• Patient intubated
• Femur fracture reduced and immobilized; pelvic stabilizing
device applied
• 500 mL warmed crystalloid and 1 unit unmatched pRBCs IV
• Vital signs: HR 97; BP 110/64; RR 24; O2 sat 96%
• Patient begins to respond to verbal stimuli, opens eyes, and tries
to brush away your hands
Initial Assessment and Management
ppnunsong yg primary dow - chest ✗
ray
( ppluic ✗-
ray
Course Overview
36 of 21
1
Discussion Questions:
1. What additional adjuncts and
treatments would you order at this
time?
2. When should the transfer occur and
what tests are necessary before
transferring the patient?
Initial Assessment and Management
Case Details
• Patient intubated
• Femur fracture reduced and
immobilized; pelvic stabilizing
device applied
• 500 mL warmed crystalloid
and 1 unit unmatched pRBCs
IV
• Vital signs: HR 97; BP 110/64;
RR 24; O2 sat 96%
• Patient begins to respond to
verbal stimuli, opens eyes,
and tries to brush away your
hands
Course Overview
37 of 21
1
Case Scenario Progression
• Patient’s LOC decreases
• Patient opens his eyes to
pressure and moves away from
stimulus (normal flexion)
• Vital signs: HR 100; BP 100/60;
RR 20
• Good breath sounds bilaterally
Initial Assessment and Management
Course Overview
38 of 21
Secondary survey:
• Pupils: 5 mm, minimally reactive, L; 6
mm, reactive, R
• Laceration and soft tissue injury, L
temporal-frontal region; no active
bleeding
• L hemotympanum
• Large ecchymosis, L anterior chest
• Abdomen soft, nondistended
1 Initial Assessment and Management
f-
Syarat →
☒ stabil → ABC stab
-
→ fast⊕
pprw
Course Overview
39 of 21
1
Discussion Question:
What is your first step when a patient’s condition changes?
Initial Assessment and Management
↳ Reevaluation
Course Overview
40 of 21
1
Discussion Question:
When does the secondary survey occur, and how is it conducted?
Initial Assessment and Management
Course Overview
41 of 21
1
Case Scenario Progression
• No neurosurgery on site
• Decision: transfer patient to
another facility for definitive care
• Contact the family to give update
and obtain consent for transfer
Initial Assessment and Management
Course Overview
42 of 21
1
Discussion Questions:
1. The family insists on obtaining a CT of
the head, even though this will
significantly delay transport (the team
is ready). Do you agree and why?
2. What information should you provide
to the receiving facility?
Initial Assessment and Management
Case Details
• Patient intubated
• Femur fracture reduced
and immobilized; pelvic
stabilizing device applied
• 500 mL warmed crystalloid
and 1 unit unmatched
pRBCs IV
• Vital signs: HR 97; BP
110/64; RR 24; O2 sat 96%
• Patient begins to respond
to verbal stimuli, opens
eyes, and tries to brush
away your hands
fdrnhtas → Mist -
Mtkhonism
spring
treatment →tovaoasl tvrdakcn
Course Overview
43 of 21
1
Case Scenario Conclusion
The patient is transferred to a
trauma center via air, and goes to
surgery for evacuation of an
intracranial hematoma.
Initial Assessment and Management
Course Overview
44 of 21
1
Any Questions?
Initial Assessment and Management
Course Overview
45 of 21
1
Review Objectives
By the end of this interactive discussion, you will be able to:
1. Explain the importance of preparation prior to trauma patient arrival.
2. Evaluate the mechanism of injury to determine the patient’s potential
injuries.
3. Identify the correct sequence of priorities for the assessment of a multiply
injured patient.
4. Apply the principles of the primary and secondary surveys to the assessment
of a multiply injured patient.
5. Discuss the importance of reevaluating a patient who is not responding
appropriately to initial resuscitation and management.
6. Recognize patients who require transfer to another facility for definitive
management.
Initial Assessment and Management
Course Overview
46 of 21
Key Learning Points
1. The initial management of the injured patient requires:
• coordination with prehospital providers
• preparation for receiving the patient
• anticipation of injuries based on the mechanism of injury
2. The evaluation of all trauma patients follows a precise algorithm.
3. Patients who exceed the capability of the institution should be
identified rapidly and process for transfer begun.
4. Evaluate the patient according to priority using the ABCDEs.
1 Initial Assessment and Management
Course Overview
47 of 21
1 Initial Assessment and Management
Video 2
Course Overview
48 of 21
1 Initial Assessment and Management
Video 3
Course Overview
49 of 21
2 Airway and Ventilatory Management
The earliest priorities in managing the injured patient are to
ensure an intact airway and recognize a compromised airway.
pg
[
Korma
balk
→ obspruasi
]
Rppvaluasi
↓
[ Suara Sevak
{⊕¥%Éah momaxai dat bantu nafas protrusion
"
"'
Sam poi
to -both
④ ada Cpdera
Objectives
By the end of this interactive discussion, you will be able to:
1. Identify different clinical situations in which airway compromise is likely to occur.
2. Recognize the signs and symptoms of acute airway compromise in a trauma case
scenario.
3. Determine factors that may lead to a difficult airway.
4. Apply the ATLS airway algorithm to a case scenario involving a patient with a difficult
airway.
5. Define the term definitive airway.
2 Airway and Ventilatory Management
→ tubp dalam trachea dgn baton
dlkoimbongkan
Case Scenario
43-year-old obese- restrained driver lost
control of his small vehicle while traveling at
a high speed on an icy road; crashed driver’s
side into a large tree.
None reported
Patient combative during extrication
Spinal motion restricted on long spine
board; C collar; bag-mask ventilation
M
I
S
T
2 Airway and Ventilatory Management
Discussion Questions:
1. What aspects of the reported mechanism
of injury present a risk of airway
compromise?
2. Which clinical findings suggest(s) potential
airway compromise?
Case Details
M
43-year-old obese- restrained
driver lost control of his small
vehicle while traveling at high
speed on an icy road;
crashed driver’s side into a
large tree.
None reported
Patient combative during
extrication
Spinal motion restricted on
long spine board; C collar;
bag- mask ventilation
I
S
T
2 Airway and Ventilatory Management
Multitrauma → maxi/ofaaal ,
caracal trauma ,
alcohol
combative Cmplawon )
Discussion Questions:
3. How do you know if the patient’s airway is
patent?
4. What are some patient factors that may
contribute to a difficult airway in this
patient?
Case Details
M
43-year-old obese- restrained
driver lost control of his small
vehicle while traveling at high
speed on an icy road;
crashed driver’s side into a
large tree.
None reported
Patient combative during
extrication
Spinal motion restrictedon
long spine board; C collar;
bag- mask ventilation
I
S
T
2 Airway and Ventilatory Management
Komunikass dgn balk → ✗ blsa → 19 mungklna airway④
Stahl
Obpshtas & trauma
Discussion Questions:
5. Are there additional factors that may be
present in other trauma patients?
6. How might we predict a difficult airway?
Case Details
M
43-year-old obese- restrained
driver lost control of his small
vehicle while traveling at high
speed on an icy road;
crashed driver’s side into a
large tree.
None reported
Patient combative during
extrication
Spinal motion restricted on
long spine board; C collar;
bag- mask ventilation
I
S
T
2 Airway and Ventilatory Management
Extreme agp ,
☆ Pdcatric i
Brosnanf
↳mon
~
Case Scenario Progression
On arrival:
• Patient lethargic
• Asymmetric chest expansion
• Shallow breathing, O2 sat 82%
• Significant facial injuries, L chest bruising
You are attempting to assist his ventilation with bag-mask ventilation.
2 Airway and Ventilatory Management
Discussion Questions:
1. What are the symptoms of inadequate
ventilation?
2. What are the signs of inadequate
ventilation?
3. What are your next steps in
management?
4. What equipment is required?
• Patient lethargic
• Asymmetric chest expansion
• Shallow breathing, O2 sat 82%
• Significant facial injuries, L chest
bruising
• You are attempting to assist his
ventilation with bag-mask
ventilation.
Case Details
2 Airway and Ventilatory Management
bpbaslcan salon natas → A da -
haha →
mug,
f
Case Scenario Progression
• Patient now unconscious
• Vital signs: systolic BP 100; HR 120
• Upper airway suctioned, some bloody sputum cleared
• Teeth and facial bones intact
• Oral airway inserted and bag-mask ventilation continued with 100%
oxygen. O2 sat improves to 93%
• You decide to intubate the patient
2 Airway and Ventilatory Management
Discussion Questions:
1. What constitutes a definitive airway?
2. How do you prepare for this
intubation?
3. What adjuncts might be used during
intubation?
Case Details
• Patient now unconscious
• Vital signs: systolic BP 100; HR 120
• Upper airway suctioned, some bloody
sputum cleared
• Teeth and facial bones intact
• Oral airway inserted and bag-mask
ventilation continued with 100%
oxygen. O2 sat improves to 93%
• You decide to intubate the patient
2 Airway and Ventilatory Management
Case Scenario Progression
• You have difficulty intubating the patient.
• LMA is inserted, but it is difficult to get an adequate seal.
• An attempt with a gum elastic bougie is successful.
• The patient oxygenation improves.
2 Airway and Ventilatory Management
Discussion Questions:
1. When is a surgical airway indicated? Should
one have been performed now
2. Is there anything that may have been
considered for intubation prior to the
patient’s decompensation?
Case Details
• You have difficulty intubating
the patient.
• LMA is inserted, but it is
difficult to get an adequate
seal.
• Successful drug-assisted
intubation performed using a
gum elastic bougie.
• The patient oxygenation
improves.
2 Airway and Ventilatory Management
extreme trauma meal Cofaacal
Case Scenario Progression
• Following successful drug-assisted intubation:
• Vital signs a few minutes following intubation: HR 130; BP 90/30; O2
sat 70%.
• Breath sounds are diminished on the L side.
2 Airway and Ventilatory Management
- Evalbass Ett → false
-
trauma - homothorax
l
pneumothorax
Discussion Questions:
1. What are the possible causes of this
patient’s deterioration?
2. How can you diagnose and treat these
possible causes?
Case Details
• Successful drug-assisted
intubation performed
• Vital signs a few minutes
following intubation: HR 130;
BP 90/30; O2 sat 70%.
• Breath sounds are diminished
on the L side.
2 Airway and Ventilatory Management
Somnath
?
M
{
look 5
Mkt hiding
Phngnmbangon dada
gurgling
Menial airway {
""m →
Suara haters abnormal
✗snoomg
stridor
HM
% hcembusan
ragas
Case Scenario Conclusion
• Right mainstem intubation is discovered and readjusted.
• Patient’s vital signs return to normal.
• Primary survey is completed.
2 Airway and Ventilatory Management
Any Questions?
2 Airway and Ventilatory Management
Objectives
By the end of this interactive discussion, you will be able to:
1. Identify different clinical situations in which airway compromise is likely to occur.
2. Recognize the signs and symptoms of acute airway compromise in a trauma case
scenario.
3. Determine factors that may lead to a difficult airway.
4. Apply the ATLS airway algorithm to a case scenario involving a patient with a difficult
airway.
5. Define the term definitive airway.
2 Airway and Ventilatory Management
Timon
Key Learning Points
1. One of earliest priorities is recognizing a compromised airway.
2. All trauma patients should receive supplemental oxygen.
3. Risk of airway compromise and difficult airway management can be
predicted.
4. Alterations in mental status (agitation, combativeness, confusion, or
obtundation) may indicate the need for airway management.
5. A definitive airway (cuffed tube in trachea below vocal cords) should be
obtained in cases of airway compromise.
2 Airway and Ventilatory Management
Skill Station :
Basic and Advanced Airway Management
Nasopharyngeal Airway
Airway Video 1
Oropharyngeal Airway
Airway Video 2
Bag-Mask Ventilation
Airway Video 3
Laryngeal Mask Airway
Airway Video 4
Oral Endotracheal Intubation
Airway Video 5
Gum Elastic Bougie
Airway Video 6
Needle Cricothyrotomy
Airway Video 7
Surgical Cricothyrotomy
Airway Video 8
Infant Endotracheal Intubation
Airway Video 9
79 of 17
Shock
3
The first step in the initial management of shock is to recognize its presence.
The diagnosis of shock is based on clinical recognition of the presence of
inadequate tissue perfusion and oxygenation.
80 of 17
Shock
3
Objectives
By the end of this interactive discussion, you will be able to:
1. Apply the ATLS principles to the management of a trauma patient with shock.
2. Recognize the signs and symptoms of shock.
3. Evaluate a patient case scenario to determine the possible causes of shock.
4. Discuss the changes that may be seen on initial investigations of a patient with shock.
5. Evaluate the efficacy of initial fluid management of a patient in shock.
6. Discuss the impact of special patient factors on the management of shock.
81 of 17
Shock
3
Case Scenario
80-year-old male, unrestrained
passenger in a low-speed MVC
None reported
Patient confused
Brought to ED by paramedics
M
I
S
T
82 of 17
Shock
3
Discussion Questions:
1. What are the possible reasons for the
patient’s confusion?
2. What additional scene information would
be helpful to obtain from the prehospital
providers that could help you differentiate
the causes of his confusion?
Case Details
M
80-year-old male,
unrestrained
passenger in a low-
speed MVC
None reported
Patient confused
Brought to ED by
paramedics
I
S
T
83 of 17
Shock
3
Case Scenario Progression
• Vital signs: BP 100/70; HR 100; RR 20
• Patient on a stretcher, receiving IV fluids
84 of 17
Shock
3
Discussion Questions:
1. Based on the information given, is this
patient in shock? What additional
information is needed to help
determine this?
2. What vital signs and laboratory studies
support the diagnosis of shock?
3. Can a single vital sign or laboratory
result diagnose shock?
• Vital signs: BP 100/70;
HR 100; RR 20
• Patient on a stretcher,
receiving IV fluids
Case Details
85 of 17
Shock
3
Discussion Questions:
4. What is the most common
cause of shock in a trauma
patient?
5. What types of soft tissue or
bony injuries might result in
shock?
• Vital signs: BP 100/70;
HR 100; RR 20
• Patient on a stretcher,
receiving IV fluids
Case Details
86 of 17
Shock
3
Case Scenario Progression
• Two large-bore IVs placed, 1 L crystalloid bolus given
• Vital signs post treatment: BP 98/77; HR 80
• The patient remains confused and unable to give his medical history
87 of 17
Shock
3
Discussion Questions:
1. What adjuncts should be considered to
determine the cause of shock (if not
already done)?
2. How should further resuscitation
proceed?
3. How will you continue to monitor this
patient’s ongoing response to fluid
resuscitation?
Case Details
• Two large-bore IVs placed,
1 L crystalloid bolus given
• Vital signs post treatment:
BP 98/77; HR 80
• The patient remains
confused and unable to
give his medical history
88 of 17
Shock
3
Case Scenario Progression
• Chest x-ray shows multiple rib fractures, large R hemothorax
• The patient’s family arrives and provides history:
o coronary artery disease
o coronary artery stenting 1 year ago
o Meds: beta blocker, clopidogrel (Plavix), and aspirin
89 of 17
Shock
3
Discussion Questions:
1. How do the beta blockers affect this
patient’s presentation and response to
interventions?
2. What management concerns are presented
by the antiplatelet agents the patient is
taking?
3. What medical condition could further
impact the patient’s response to shock?
Case Details
• Chest x-ray shows multiple
rib fractures, large R
hemothorax
• Patient history:
o coronary artery disease
o coronary artery stenting
1 year ago
o Meds: beta blocker,
clopidogrel (Plavix), and
aspirin
90 of 17
Shock
3
Case Scenario Conclusion
• Platelet transfusion initiated.
• R chest tube is placed 750 mL of blood obtained
• Subsequent chest tube output is 50 mL/2 hours
91 of 17
Shock
3
Any Questions?
92 of 17
Shock
3
Review Objectives
By the end of this interactive discussion, you will be able to:
1. Apply the ATLS principles to the management of a trauma patient with shock.
2. Recognize the signs and symptoms of a trauma patient in shock.
3. Evaluate a patient case scenario to determine the possible causes of shock.
4. Discuss the changes that may be seen on initial investigations of a patient with shock.
5. Evaluate the efficacy of initial fluid management of a patient in shock.
6. Discuss the impact of special patient factors on the management of shock.
93 of 17
Shock
3
Key Learning Points
1. Hemorrhage is the most common cause of shock after injury.
2. No single laboratory test and no single vital sign on its own can diagnose shock.
3. Massive blood loss may produce only minimal acute decrease in hemoglobin or
hematocrit.
4. Major soft tissue injuries and fractures can be associated with significant hemorrhage.
5. The patient’s response to initial fluid therapy will help guide subsequent therapy.
6. A variety of special conditions may affect the patient’s response to shock and the
management of it (e.g., age, medication use).
94 of 17
Shock
3
95 of 17
Shock
3
Thoracic injury is common in polytrauma patients and can be life-threatening,
especially if not promptly identified and treated during the primary survey.
4 Thoracic Trauma
96 of 17
Shock
3
Objectives
By the end of this interactive discussion, you will be able to:
1. Apply the ATLS principles to the management of a patient with thoracic trauma.
2. Recognize the important life-threatening injuries in a patient with thoracic trauma.
3. Evaluate the case scenario of a patient with thoracic trauma to identify immediate life-
threatening injuries.
4. Discuss the clinical findings and adjunctive studies that may be useful during the
secondary survey in a patient with thoracic trauma.
4 Thoracic Trauma
97 of 17
Shock
3
Case Scenario
27-year-old male unrestrained driver in
high-speed, frontal-impact MVC
Airway patent, obvious respiratory
distress
BP 90/50; HR 110; RR 36; and GCS 14
None reported
M
I
S
T
4 Thoracic Trauma
98 of 17
Shock
3
Discussion Questions:
1. What life-threatening injuries might one
discover in the primary survey that could
account for the patient’s clinical status?
2. What are the next steps in the evaluation
and treatment of this patient?
Case Details
M
27-year-old male
unrestrained driver in
high-speed, frontal-impact
MVC
Airway patent, obvious
respiratory distress
BP 90/50; HR 110; RR 36;
and GCS 14
None reported
I
S
T
4 Thoracic Trauma
99 of 17
Shock
3
Discussion Questions:
1. What are the immediately life-threatening
thoracic injuries involving:
• Airway
• Breathing
• Circulation
Case Details
M
27-year-old male
unrestrained driver in
high-speed, frontal-impact
MVC
Airway patent, obvious
respiratory distress
BP 90/50; HR 110; RR 36;
and GCS 14
None reported
I
S
T
4 Thoracic Trauma
100 of 17
Shock
3
Case Scenario Progression
On arrival to ED:
• Patient complains of profound shortness of breath, asking to sit up
• O2 sat 89%.
• Cervical collar in place
• Trachea deviated to left
• Breath sounds absent on left
• Heart sounds normal
• Left chest wall crepitus
4 Thoracic Trauma
101 of 17
Shock
3
Discussion Questions:
1. What is this patient’s most likely
diagnosis?
2. What is the appropriate
technique to alleviate this
patient’s condition?
• Patient complains of profound
shortness of breath, asking to sit
up
• O2 sat 89%.
• Cervical collar in place
• Trachea deviated to left
• Breath sounds absent on left
• Heart sounds normal
• Left chest wall crepitus
Case Details
4 Thoracic Trauma
102 of 17
Shock
3
Case Scenario Progression
• Needle decompression performed, no rush of air
• Vital signs unchanged
4 Thoracic Trauma
103 of 17
Shock
3
Discussion Question:
What alternative management strategy
can you use?
Case Details
• Needle decompression
performed, no rush of air
• Vital signs unchanged
4 Thoracic Trauma
104 of 17
Shock
3
Discussion Question:
What are the differences in clinical
presentation between a tension
pneumothorax and an open pneumothorax?
4 Thoracic Trauma
105 of 17
Shock
3
Case Scenario Progression
• Finger thoracostomy performed
• Vital signs: RR 28; HR 110; BP 100/60
• Thoracostomy tube placed, 600 mL dark blood drained
• Two large-bore IV lines established, isotonic fluid given
• Type and crossmatch requested
• Chest x-ray shows:
o obscured L diaphragm
o multiple L rib fractures
o wide mediastinum
o pulmonary contusion
4 Thoracic Trauma
106 of 17
Shock
3
Discussion Questions:
1. What are the indications for operation in
a patient with traumatic hemothorax?
2. What resuscitative measures should be
undertaken in a patient with massive
hemothorax?
4 Thoracic Trauma
107 of 17
Shock
3
Discussion Questions:
3. What other potential life threats
might exist in a patient with this
mechanism? For each, what would
the clinical presentation/findings
and the appropriate treatment be?
4. What test would you perform to
make the diagnosis during the
secondary survey?
• Finger thoracostomy performed
• Vital signs: RR 28; HR 110; BP 100/60
• Thoracostomy tube placed, 600 mL dark
blood drained
• Two large-bore IV lines established,
isotonic fluid given
• Type and crossmatch requested
• Chest x-ray shows:
• obscured L diaphragm
• multiple L rib fractures
• wide mediastinum
• pulmonary contusion
Case Details
4 Thoracic Trauma
108 of 17
Shock
3
Potential Life Threat Clinical
Presentation/Findings
Treatment Pitfalls
Simple Pneumothorax +/- shortness of breath No
hypotension
Diagnosis by chest x-ray
Chest tube drainage Could become tension
pneumothorax if untreated
Hemothorax Dullness to percussion
Diagnosis by chest x-ray
Chest tube drainage Could become massive
hemothorax
Flail Chest and Pulmonary
Contusion
May see paradoxical movement
of chest wall More commonly
presents with pain and poor
respiratory excursions
Oxygen
Analgesia
Intubation if necessary
Progressive respiratory failure
Blunt Cardiac Injury ECG changes Cardiac monitoring
Therapy based on clinical
status
At risk for clinically significant
dysrhythmias
Traumatic Aortic Disruption May be asymptomatic
Multiple possible radiographic
findings
Endovascular or open surgical
repair
Blood pressure control
important prior to definitive
therapy
Traumatic Diaphragm Injury Respiratory distress
Obscured left diaphragm
border
Evidence of abdominal viscera
in chest
Operative repair Concomitant pulmonary
contusion may mask diaphragm
injury
Esophageal injury Chest pain; mediastinal air on
imaging; crepitus
delayed fever
Operative repair Delayed diagnosis
4 Thoracic Trauma
109 of 17
Shock
3
Case Scenario Progression
• 250 mL of IV fluids given
• Vital signs: BP 110/70; HR 110; RR 18
• O2 sat 91% on nonrebreather mask
4 Thoracic Trauma
110 of 17
Shock
3
Discussion Question:
Which of the following treatments is best for pulmonary
contusion/flail chest?
A. Beta blockers
B. Massive fluid resuscitation
C. Immediate nebulizer treatment
D. Supplemental oxygen, pain control, and recognition of the
potential for respiratory failure
4 Thoracic Trauma
111 of 17
Shock
3
Case Scenario Conclusion
Your institution has the capability to care for this patient, and you order
a CT scan for further evaluation.
4 Thoracic Trauma
112 of 17
Shock
3
Case Scenario #2
25-year-old male, high-speed MVC
Awake and responds to questions, complaining of chest pain and
shortness of breath, gurgling sounds L lung base.
BP 102/76; HR 134; O2 sat 93% on oxygen by face mask
Chest x-ray
M
I
S
T
4 Thoracic Trauma
113 of 17
Shock
3
Discussion Questions:
1. What abnormalities do you note
on the chest film?
4 Thoracic Trauma
114 of 17
Shock
3
Discussion Questions:
2. What is the treatment for a
traumatic diaphragm injury?
3. Aside from the tracheal deviation to
the right seen on the chest film,
what other signs and x-ray findings
might one see in a patient with
traumatic aortic disruption?
Case Details
M
25-year-old male, high-speed
MVC
Awake and responds to questions,
complaining of chest pain and
shortness of breath, gurgling
sounds L lung base.
BP 102/76; HR 134; O2 sat 93% on
oxygen by face mask
Chest x-ray
I
S
T
4 Thoracic Trauma
115 of 17
Shock
3
Case Scenario Progression
CT scan shows blunt aortic injury
4 Thoracic Trauma
116 of 17
Shock
3
Discussion Questions:
1. What is the expected hemodynamic impact of
a contained aortic disruption from blunt
trauma?
2. If a patient with a known contained aortic
disruption from blunt trauma becomes
hypotensive, what should you consider?
3. What therapeutic steps should a clinician
consider when managing a traumatic aortic
disruption?
Case Details
• CT scan shows blunt aortic
injury
4 Thoracic Trauma
117 of 17
Shock
3
Case Scenario Conclusion
• Discussion with anesthesiology re: management of the patient’s
aortic injury
• Patient undergoes repair of his diaphragmatic injury via laparotomy
• The following day, he undergoes endovascular repair of his aortic
injury
• He does well after 2 weeks in the ICU and is discharged home
4 Thoracic Trauma
118 of 17
Shock
3
Any Questions?
4 Thoracic Trauma
119 of 17
Shock
3
Review Objectives
By the end of this interactive discussion, you will be able to:
1. Apply the ATLS principles to the management of a patient with thoracic trauma.
2. Recognize the important life-threatening injuries in a patient with thoracic trauma.
3. Evaluate the case scenario of a patient with thoracic trauma to identify immediate life-
threatening injuries.
4. Discuss the clinical findings and adjunctive studies that may be useful during the
secondary survey in a patient with thoracic trauma.
4 Thoracic Trauma
120 of 17
Shock
3
Key Learning Points
1. It is important to recognize thoracic life-threatening problems in polytrauma
patients.
2. Most immediate thoracic life-threatening problems can be recognized without
special testing and may be treated with:
• airway control
• decompression and/or
• fluid resuscitation
3. Potential life-threatening problems can become immediate life-threatening
problems if untreated (e.g., a simple pneumothorax can become a tension
pneumothorax).
4 Thoracic Trauma
Skill Station :
Breathing
122 of 17
Shock
3
Breathing Video 1 (paradoxical chest wall movement)
Respiratory Distress
123 of 17
Shock
3
Breathing Video 2
Chest Tube
124 of 17
Shock
3
Breathing Video 3
Respiratory Distress (Child)
125 of 17
Shock
3 Chest Tube (Child)
Breathing Video 4
126 of 17
Shock
3 Chest Tube (Child)
Breathing Video 5
127 of 17
Shock
3
Breathing Video 6
Dressing on a Sucking Chest Wound
5 Abdominal and Pelvic Trauma Tenth Edition
129 of 17
Shock
3
When uncontrolled or unrecognized, blood loss from abdominal
and pelvic injuries can result in preventable death.
5 Abdominal and Pelvic Trauma
130 of 17
Shock
3
Objectives
By the end of this interactive discussion, you will be able to:
1. Identify the anatomic regions of the abdomen that are critical in assessing and managing trauma
patients.
2. Discuss the risk for abdominal and pelvic injuries based on the mechanism of injury.
3. Identify patients who require surgical consultation and possible surgical and/or catheter-based
intervention.
4. Determine appropriate diagnostic procedures to ascertain if a patient has ongoing hemorrhage and/or
other injuries that can cause delayed morbidity and mortality.
5. Formulate an acute management plan for abdominal and pelvic injuries utilizing a case scenario.
6. Discuss the importance of early identification and emergent management of pelvic hemorrhage.
5 Abdominal and Pelvic Trauma
131 of 17
Shock
3
Case Scenario
28-year-old male, helmeted motorcyclist,
high-speed collision, head-on into the side
of a vehicle that pulled out in front of him
Patient reports brief loss of consciousness,
complains of pain in chest, abdomen, and
pelvis
BP 100/75; HR 115; RR 20, and GCS 15
Backboard and c-collar
M
I
S
T
5 Abdominal and Pelvic Trauma
132 of 17
Shock
3
Discussion Questions:
1. What are your priorities for
management of this patient?
2. What is your interpretation of
the vital signs?
3. What is your initial therapy for
this patient?
Case Details
M
28-year-old male, helmeted
motorcyclist, high-speed collision,
head-on into the side of a vehicle that
pulled out in front of him
Patient reports brief loss of
consciousness, complains of pain in
chest, abdomen, and pelvis
BP 100/75; HR 115; RR 20, and GCS
15
Backboard and c-collar
I
S
T
5 Abdominal and Pelvic Trauma
133 of 17
Shock
3
Case Scenario Progression
• EMS reports:
o Patient found 10 feet (3 meters) from his motorcycle
o Patient lying on R side, wearing a helmet
o Had been travelling at 45 mph (70 kph)
• Patient reports:
o Hard R sided landing, brief LOC
o No allergies, no previous medical history or current medications
5 Abdominal and Pelvic Trauma
134 of 17
Shock
3
Discussion Question:
1. Based on the reported mechanism
of injury, what intra-abdominal
and/or pelvic injury is the patient
likely to have sustained?
• Patient found 10 feet (3 meters) from his
motorcycle
• Patient lying on R side, wearing a helmet
• Had been travelling at 45 mph (70 kph)
• Hard R sided landing, brief LOC
• No allergies, no previous medical history
or current medications
Case Details
5 Abdominal and Pelvic Trauma
135 of 17
Shock
3
Discussion Questions:
1. How would the risk of intra-
abdominal injury change if the
patient described striking the
handlebar into the epigastrium?
2. How would the risk of intra-
abdominal injury change if a
penetrating injury was observed?
• Patient found 10 feet (3 meters) from his
motorcycle
• Patient lying on R side, wearing a helmet
• Had been travelling at 45 mph (70 kph)
• Hard R sided landing, brief LOC
• No allergies, no previous medical history
or current medications
Case Details
5 Abdominal and Pelvic Trauma
136 of 17
Shock
3
Case Scenario Progression
On examination:
• Right-sided lower chest tenderness
• Contusions, R chest, abdomen, and flank
• Tender R upper quadrant, R flank, and suprapubic region
• Pain on palpation of the anterior pelvis
• No blood at the urethral meatus
• Rectal examination is normal.
• Vital signs following 500 ml of crystalloid solution: BP 110/75; HR 100; RR 20;
GCS 15
5 Abdominal and Pelvic Trauma
137 of 17
Shock
3
Discussion Questions:
1. How should you assess the abdomen
and pelvis for injury and as potential
sources of bleeding?
2. Based on your knowledge of anatomy,
the mechanism of injury, and this
patient’s physical examination, what
abdominal and/or pelvic injuries are
most likely?
3. Is a FAST exam indicated at this stage?
• Right-sided lower chest tenderness
• Contusions, R chest, abdomen, and flank
• Tender R upper quadrant, R flank, and
suprapubic region
• Pain on palpation of the anterior pelvis
• No blood at the urethral meatus
• Rectal examination is normal.
• Vital signs following 500 ml of crystalloid
solution: BP 110/75; HR 100; RR 20; GCS 15
Case Details
5 Abdominal and Pelvic Trauma
138 of 17
Shock
3
Discussion Question:
4. If this patient were female, what other
examination would be relevant?
5. What radiological investigations would
be appropriate to arrange now?
• Right-sided lower chest tenderness
• Contusions, R chest, abdomen, and flank
• Tender R upper quadrant, R flank, and
suprapubic region
• Pain on palpation of the anterior pelvis
• No blood at the urethral meatus
• Rectal examination is normal.
• Vital signs following 500 ml of crystalloid
solution: BP 110/75; HR 100; RR 20; GCS 15
Case Details
5 Abdominal and Pelvic Trauma
139 of 17
Shock
3
Case Scenario Progression
• Your institution has full surgical and radiological capabilities
• Abdominal CT: grade III liver injury, R rib fractures, bilateral pelvic
rami fractures
• Surgical consultation obtained
• Vital signs: BP normal; HR 100
• A total of 1.0 L of crystalloid solution given
5 Abdominal and Pelvic Trauma
140 of 17
Shock
3
Discussion Question:
1. Is emergent laparotomy warranted in
this patient?
2. What clinical changes in this patient
would indicate the need for operation,
other therapies, or additional
investigations regarding the abdominal
and pelvic injuries?
3. How would your management change if
the CT scan identified contrast
extravasation suggesting bleeding in the
pelvis?
• Your institution has full surgical and
radiological capabilities
• Abdominal CT: grade III liver injury,
R rib fractures, bilateral pelvic rami
fractures
• Surgical consultation obtained
• Vital signs: BP normal; HR 100
• A total of 1.0 L of crystalloid
solution given
Case Details
5 Abdominal and Pelvic Trauma
141 of 17
Shock
3
Case Scenario Conclusion
• Emergent laparotomy not required; nonoperative management
undertaken
• Patient admitted to ICU for monitoring, pain control, and respiratory
care
• Hemodynamics normal over 24 hours, transferred to the ward
• Physical therapy for the pelvic fractures
• Patient discharged home on day 6
5 Abdominal and Pelvic Trauma
142 of 17
Shock
3
Any Questions?
5 Abdominal and Pelvic Trauma
143 of 17
Shock
3
Review Objectives
By the end of this interactive discussion, you will be able to:
1. Identify the anatomic regions of the abdomen that are critical in assessing and managing trauma
patients.
2. Discuss the risk for abdominal and pelvic injuries based on the mechanism of injury.
3. Identify patients who require surgical consultation and possible surgical and/or catheter-based
intervention.
4. Determine appropriate diagnostic procedures to ascertain if a patient has ongoing hemorrhage and/or
other injuries that can cause delayed morbidity and mortality.
5. Formulate an acute management plan for abdominal and pelvic injuries utilizing a case scenario.
6. Discuss the importance of early identification and emergent management of pelvic hemorrhage.
5 Abdominal and Pelvic Trauma
144 of 17
Shock
3
Key Learning Points
1. Mechanism of injury is critical when considering abdominal and/or pelvic injury.
2. Thorough examinations of the chest, abdomen, and pelvis (anterior, lateral, posterior,
and perineum) are required to avoid missing significant injuries.
3. Appropriate diagnostic procedures should be employed.
4. Surgical intervention is assessed via clinical findings and the patient’s response to
management.
5. Early identification and emergent management of pelvic hemorrhage can be
lifesaving.
5 Abdominal and Pelvic Trauma
Skill Station :
Circulation
146 of 17
Shock
3
Circulation Video 1
Packing an Open Wound
147 of 17
Shock
3
Circulation Video 2
Tourniquet
148 of 17
Shock
3
Circulation Video 3
Splint Traction
149 of 17
Shock
3 Splint Traction
before traction after traction
150 of 17
Shock
3
Circulation Video 4
Intraosseous
151 of 17
Shock
3
Circulation Video 5
Pelvic Binder
152 of 17
Shock
3 Pelvic Binder
before binder after binder
Foto : pelvic diasthesis
153 of 17
Shock
3
Circulation Video 6
Central Venous Catheter
154 of 17
Shock
3
Circulation Video 7
FAST
156 of 17
Shock
3
Circulation Video 9
Pericardiocentesis
157 of 17
Shock
3
158 of 17
Shock
3
The primary goal of treatment for patients with suspected
traumatic brain injury is to prevent secondary brain injury.
6 Head Trauma
159 of 17
Shock
3
Objectives
By the end of this interactive discussion, you will be able to:
1. Recognize the GCS score that corresponds to a severe head injury and indicates a comatose
patient.
2. Identify the different types of intracranial bleeding seen on CT that are associated with
traumatic brain injury.
3. Discuss the role of supplemental oxygen and systolic blood pressure maintenance in limiting
secondary brain injury.
4. Describe the management of intracranial hypertension associated with the mass effect of
blood or brain swelling.
5. Discuss the indications for early, rapid transfer to a center equipped to manage a patient with
brain injury.
6 Head Trauma
160 of 17
Shock
3
Case Scenario
23-year-old male, fell from bicycle, hitting head on curb; no helmet
10 cm laceration to the L temporal-parietal region
Initially able to say his name. HR 115; BP 100/60; O2 sat 88%; GCS 12 (E3V3M6)
Two hours after transport to local hospital, patient has sonorous respirations; HR 120;
BP 100/70; GCS 6 (E2V1M3)
IV cannulas in situ, O2 via nasal prongs, 200mLs crystalloid infused
M
I
S
T
6 Head Trauma
161 of 17
Shock
3
Discussion Question:
1. What are the initial
priorities in the
management of this
patient?
2. What are the signs that
the patient’s injury is
progressing?
Case Details
M
23-year-old male, fell from bicycle, hitting head on
curb; no helmet
10 cm laceration to the L temporal-parietal region
Initially able to say his name. HR 115; BP 100/60;
O2 sat 88%; GCS 12 (E3V3M6)
Two hours after transport to local hospital, patient
has sonorous respirations; HR 120; BP 100/70; GCS
6 (E2V1M3)
IV cannulas in situ, O2 via nasal prongs, 200mLs
crystalloid infused
I
S
T
6 Head Trauma
162 of 17
Shock
3
Case Scenario Progression
• Patient intubated
• Given 1 L normal saline
• Vital signs: HR 100; BP 100/70; O2
Sat 94%
6 Head Trauma
163 of 17
Shock
3
Discussion Question:
1. How do you monitor this patient’s
neurological status?
2. What other injuries and physical exam
findings may suggest cranial and
intracranial injury?
• Patient intubated
• Given 1 L normal saline
• Vital signs: HR 100;BP 100/70;
O2 Sat 94%
Case Details
6 Head Trauma
164 of 17
Shock
3
Case Scenario Progression
• Head, c-spine and abdominal CTs performed.
• Head CT: temporal bone fracture, epidural hematoma, 1 cm of midline shift
• C-spine normal
6 Head Trauma
165 of 17
Shock
3
Discussion Questions:
1. What types of intracranial
hemorrhage can be identified on
CT scan?
6 Head Trauma
166 of 17
Shock
3
Discussion Question:
2. What CT scan findings are indicative
of severe head injury that may require
intervention?
• Head, c-spine and abdominal
CTs performed.
• Head CT: temporal bone
fracture, epidural hematoma,
1 cm of midline shift
• C-spine normal
Case Details
6 Head Trauma
167 of 17
Shock
3
Case Scenario Progression
• Thoracoabdominal CT scan normal
• Initial management includes:
o elevating the head of bed
o sedation with short-acting medications
o frequent neurological examinations
6 Head Trauma
168 of 17
Shock
3
Discussion Question:
1. What are the initial management
options for this patient with
severe brain injury and how do
these differ from mild and
moderate brain injury?
2. What are the indications for
transferring a patient with a head
injury to a center with a higher
level of care?
• Thoracoabdominal CT scan normal
• Initial management includes:
• elevating the head of bed
• sedation with short-acting
medications
• frequent neurological
examinations
Case Details
6 Head Trauma
169 of 17
Shock
3
Case Scenario Progression
• Neuro exam shows progression to extensor
posturing.
• Repeat CT scan shows new subdural hematoma
with associated mass effect and midline shift.
• Herniation appears imminent without treatment.
• Patient requires a higher level of care and rapid
transfer to neurosurgeon.
6 Head Trauma
170 of 17
Shock
3
Discussion Question:
What are the initial treatment
options that may protect the
brain from ongoing swelling?
• Neuro exam shows progression to
extensor posturing.
• Repeat CT scan shows new subdural
hematoma with associated mass effect
and midline shift.
• Herniation appears imminent without
treatment.
• Patient requires a higher level of care
and rapid transfer to neurosurgeon.
Case Details
6 Head Trauma
171 of 17
Shock
3
Case Scenario Conclusion
• Neurosurgeon recommends 0.5 g/kg mannitol and adjusting PaCO2 to 30
to 35 mm Hg.
• Patient is immediately transported for emergency craniotomy.
• Patient underwent successful evacuation of his intracranial hematoma.
• He was discharged to a rehabilitation center for ongoing therapy.
6 Head Trauma
172 of 17
Shock
3
Any Questions?
6 Head Trauma
173 of 17
Shock
3
Review Objectives
By the end of this interactive discussion, you will be able to:
1. Recognize the GCS score that corresponds to a severe head injury and indicates a comatose
patient.
2. Identify the different types of intracranial bleeding seen on CT that are associated with
traumatic brain injury.
3. Discuss the role of supplemental oxygen and systolic blood pressure maintenance in limiting
secondary brain injury.
4. Describe the management of intracranial hypertension associated with the mass effect of
blood or brain swelling.
5. Discuss the indications for early, rapid transfer to a center equipped to manage a patient with
brain injury.
6 Head Trauma
174 of 17
Shock
3
Key Learning Points
• GCS score is an objective, reproducible measurement of brain injury severity.
• GCS of 8 or less is considered severe and indicative of a comatose patient.
• Consider a CT scan of the head for any trauma patient with suspected traumatic brain
injury.
• Initial management of intracranial hypertension includes:
• elevation of the head of bed
• sedation
• selective administration of mannitol and hypertonic saline
6 Head Trauma
175 of 17
Shock
3
Key Learning Points
• Minimize secondary brain injury by:
• adequate oxygenation (supplemental oxygen)
• ensuring brain perfusion: SBP > 100 mm Hg (age 50-69) or > 110 mm Hg (15 – 49 and
older than 70)
• If no neurosurgical capability, consider early, rapid transfer
6 Head Trauma
176 of 17
Shock
3
177 of 17
Shock
3
Because spine injury can occur with both blunt and penetrating
trauma, and with or without neurological deficits, it must be
considered in all patients with multiple injuries. These patients require
restriction of spinal motion to protect the spine from further damage
until spine injury has been ruled out.
7 Spine and Spinal Cord Trauma
178 of 17
Shock
3
Objectives
By the end of this interactive discussion, you will be able to:
1. Apply the ABC principles of ATLS when assessing a patient for spine injury.
2. Identify a common mechanism and type of spinal injury.
3. Describe the typical signs and symptoms of a patient with a spinal cord injury.
4. Describe the technique and importance of documentation of a potential spinal injury.
5. Describe the appropriate initial treatment of patients with spinal injuries.
6. Determine the appropriate disposition of patients with spine trauma.
7 Spine and Spinal Cord Trauma
179 of 17
Shock
3
Case Scenario
28-year-old male fell 5 meters (16 feet)
from scaffolding, wearing hard hat;
bystander reports patient landed head-first,
with neck hyperextended
None reported
Skin warm; breathing shallow; not moving
arms or legs; Vital signs: BP 80/62; HR 58;
RR 28; GCS 15
None Reported
M
I
S
T
7 Spine and Spinal Cord Trauma
180 of 17
Shock
3
Discussion Questions:
1. What injuries has the patient
likely sustained?
2. What other mechanisms of
injury are associated with spinal
cord trauma?
3. What types of shock are
potentially present in this
patient?
Case Details
M
28-year-old male fell 5 meters (16 feet)
from scaffolding, wearing hard hat;
bystander reports patient landed head-
first, with neck hyperextended
None reported
Skin warm; breathing shallow; not
moving arms or legs; Vital signs:
BP 80/62; HR 58; RR 28; GCS 15
None Reported
I
S
T
7 Spine and Spinal Cord Trauma
181 of 17
Shock
3
Discussion Questions:
4. How should you initially manage
this patient?
5. What would you do if the
patient was not breathing?
Case Details
M
28-year-old male fell 5 meters (16 feet)
from scaffolding, wearing hard hat;
bystander reports patient landed head-
first, with neck hyperextended
None reported
Skin warm; breathing shallow; not
moving arms or legs; Vital signs:
BP 80/62; HR 58; RR 28; GCS 15
None Reported
I
S
T
7 Spine and Spinal Cord Trauma
182 of 17
Shock
3
Discussion Questions:
6. Which signs and symptoms are
concerning for a spinal injury in
this patient?
7. What other signs and symptoms
not previously mentioned may
be associated with the presence
of a spinal cord injury?
Case Details
M
28-year-old male fell 5 meters (16 feet)
from scaffolding, wearing hard hat;
bystander reports patient landed head-
first, with neck hyperextended
None reported
Skin warm; breathing shallow; not
moving arms or legs; Vital signs:
BP 80/62; HR 58; RR 28; GCS 15
None Reported
I
S
T
7 Spine and Spinal Cord Trauma
183 of 17
Shock
3
Case Scenario Progression
Peripheral neurologic exam:
• Patient unable to move or feel legs
• Patient can move fingers and wrists bilaterally
• Weak L triceps extension
• Patient unable to move R elbow
• Patient able to feel fingers and thumbs bilaterally
• Patient unable to feel inner arms above the elbows
7 Spine and Spinal Cord Trauma
184 of 17
Shock
3
Discussion Questions:
1. At what level is the suspected spinal
lesion?
2. What resources are available to assist
with determining the level of spinal
injury?
3. Why is the exam different between
the right and left upper extremities?
• Patient unable to move or feel legs
• Patient can move fingers and
wrists bilaterally
• Weak L triceps extension
• Patient unable to move R elbow
• Patient able to feel fingers and
thumbs bilaterally
• Patient unable to feel inner arms
above the elbows
Case Details
7 Spine and Spinal Cord Trauma
185 of 17
Shock
3
Discussion Questions:
4. What are important aspects of
documenting the neurologic
examination?
5. Why is it important to accurately and
thoroughly document the neurologic
examination findings?
7 Spine and Spinal Cord Trauma
186 of 17
Shock
3
Case Scenario Progression
• Patient is spinal motion restricted
• 1 L of crystalloid fluid is given
• Vital signs: BP 100/64; HR 62; RR 28; GCS 15
7 Spine and Spinal Cord Trauma
187 of 17
Shock
3
Discussion Questions:
1. What imaging would you request for
this patient?
2. What tools are available to assist with
decisions regarding spine imaging?
• Patient is spinal motion
restricted
• 1 L of crystalloid fluid is given
• Vital signs: BP 100/64; HR 62;
RR 28; GCS 15
Case Details
7 Spine and Spinal Cord Trauma
188 of 17
Shock
3
Case Scenario Progression
• Negative chest x-ray, pelvic x-ray, and FAST exam
• CT scan unavailable.
• Lateral c-spine x-ray: C6 fracture with anterior displacement.
• Patient placed in c-collar with mobility restrictions
• Vital signs: BP 80/62; HR 58; RR 28; GCS 15
7 Spine and Spinal Cord Trauma
189 of 17
Shock
3
Discussion Question:
How do you manage the patient’s
ABCs at this point?
• Negative chest x-ray, pelvic x-ray,
and FAST exam
• CT scan unavailable.
• Lateral c-spine x-ray: C6 fracture
with anterior displacement.
• Patient placed in c-collar with
mobility restrictions
• Vital signs: BP 80/62; HR 58; RR
28; GCS 15
Case Details
7 Spine and Spinal Cord Trauma
190 of 17
Shock
3
Case Scenario Progression
• Following additional 1 L of IV crystalloid, BP remains 80/50, HR 45
• RR increased to 30
• Patient complains of shortness of breath
• ABGs: PCO2 50 mm Hg
7 Spine and Spinal Cord Trauma
191 of 17
Shock
3
Discussion Questions:
1. What interventions are indicated
at this facility?
2. Does this patient require transfer?
What is the rationale for this
decision?
• Following additional 1 L of
IV crystalloid, BP remains
80/50, HR 45
• RR increased to 30
• Patient complains of
shortness of breath
• ABGs: PCO2 50 mm Hg
Case Details
7 Spine and Spinal Cord Trauma
192 of 17
Shock
3
Case Scenario Conclusion
• Patient intubated
• Systolic BP improves to 110/65 with vasopressor support
• O2 sat 98% on 30% oxygen
• Repeat FAST negative
• Transferred to definitive care, where C6 and T6 fractures identified
• Admitted to ICU
• Surgery for C6 fracture, nonoperative management of T6 fracture
• Transferred to a spinal cord rehabilitation center
7 Spine and Spinal Cord Trauma
193 of 17
Shock
3
Any Questions?
7 Spine and Spinal Cord Trauma
194 of 17
Shock
3
Review Objectives
By the end of this interactive discussion, you will be able to:
1. Apply the ABC principles of ATLS when assessing a patient for spine injury.
2. Identify a common mechanism and type of spinal injury.
3. Describe the typical signs and symptoms of a patient with a spinal cord injury.
4. Describe the technique and importance of documentation of a potential spinal injury.
5. Describe the appropriate initial treatment of patients with spinal injuries.
6. Determine the appropriate disposition of patients with spine trauma.
7 Spine and Spinal Cord Trauma
195 of 17
Shock
3
Key Learning Points
1. Attend to the life-threatening injuries identified in the primary survey while minimizing
movement of the spine.
2. Assume possible spinal injury until clinical and/or radiographic evaluation can be completed
(decision tools such as Canadian C-Spine Rules or NEXUS may be used).
3. Be as specific and accurate as possible when describing and documenting the level of neurologic
injury (ASIA tool is extremely useful).
4. High spinal cord injuries may be associated with respiratory failure and/or neurogenic shock,
which must be addressed prior to transfer.
5. Consider obtaining early consultation with a spine surgeon when a spinal injury is suspected
and/or detected.
7 Spine and Spinal Cord Trauma
Skill Station :
Disability
197 of 17
Shock
3
Case Scenario #1
A 65-year-old male is transported to the emergency department by EMS after a motor
vehicle collision while driving home at 50 km/hr (30 mph) from a tavern. He is currently
being treated for a recent deep vein thrombosis.
He has a left scalp contusion. He has a strong odor of alcohol on his breath. He is thought
to have suffered a concussion and to have alcohol intoxication.
His airway is clear, he is breathing spontaneously without difficulty, and he has no
hemodynamic abnormalities. His eyes are open. He appears confused and pushes away the
examiner’s hands when assessed for response to fingertip pressure.
He has a cervical collar in place, and motion is restricted on a long spine board.
M
I
S
T
GCS ?
198 of 17
Shock
3
Case Scenario #1
A 65-year-old male is transported to the emergency department by EMS after a motor
vehicle collision while driving home at 50 km/hr (30 mph) from a tavern. He is currently
being treated for a recent deep vein thrombosis.
He has a left scalp contusion. He has a strong odor of alcohol on his breath. He is thought
to have suffered a concussion and to have alcohol intoxication.
His airway is clear, he is breathing spontaneously without difficulty, and he has no
hemodynamic abnormalities. His eyes are open. He appears confused and pushes away the
examiner’s hands when assessed for response to fingertip pressure.
He has a cervical collar in place, and motion is restricted on a long spine board.
M
I
S
T
GCS ?
E : 4
V : 4 (?)
M : 4
GCS 12 (?)
199 of 17
Shock
3
Case Scenario #3
A 75-year-old male is brought to a rural emergency department by his family after a
ground-level fall after slipping on a water puddle.
According to his family, he had an approximate 30-second loss of consciousness. The
patient does not remember the injury event and only complains of a headache. His past
medical history is only significant for hypertension. Other than a small left frontoparietal
soft tissue contusion, there are no other gross injuries.
His vital signs are: BP 140/85, HR 70, and RR 16. Other than a small left frontoparietal soft
tissue contusion, there are no other gross injuries. He is awake, opens his eyes
spontaneously, follows commands, communicates appropriately, and does not have any
focal neurological deficits.
Triage personnel put patient in cervical collar.
M
I
S
T
GCS ?
200 of 17
Shock
3
Case Scenario #3
A 75-year-old male is brought to a rural emergency department by his family after a
ground-level fall after slipping on a water puddle.
According to his family, he had an approximate 30-second loss of consciousness. The
patient does not remember the injury event and only complains of a headache. His past
medical history is only significant for hypertension. Other than a small left frontoparietal
soft tissue contusion, there are no other gross injuries.
His vital signs are: BP 140/85, HR 70, and RR 16. Other than a small left frontoparietal soft
tissue contusion, there are no other gross injuries. He is awake, opens his eyes
spontaneously, follows commands, communicates appropriately, and does not have any
focal neurological deficits.
Triage personnel put patient in cervical collar.
I
S
T
GCS ?
E : 4
V : 5
M : 6
GCS 15
M
201 of 17
Shock
3
Disability Video 1
Helmet Removal
202 of 17
Shock
3
Disability Video 2
Logrolling Technique
203 of 17
Shock
3
CT of head : epidural hematoma
Epidural Hematoma
204 of 17
Shock
3
CT of head : subdural hematoma
Subdural Hematoma
205 of 17
Shock
3
CT of head : subarachnoid hemorrhage and hemorrhage on the left frontoparietal area
Subarachnoid Hemorrhage
206 of 17
Shock
3
CT of head : intracerebral hematoma
Intracerebral Hematoma
207 of 17
Shock
3
208 of 17
Shock
3
Injuries to the musculoskeletal system are common in trauma patients.
The delayed recognition and treatment of these injuries can result in
life-threatening hemorrhage or limb loss.
8 Musculoskeletal Trauma
209 of 17
Shock
3
Objectives
By the end of this interactive discussion, you will be able to:
1. Explain the significance of musculoskeletal injuries in patients with multiple injuries.
2. Outline the priorities of the primary survey, resuscitation and secondary survey of
patients with extremity injuries.
3. Identify the adjuncts needed in the immediate treatment of life-threatening extremity
hemorrhage.
4. Explain the principles of the initial management of limb-threatening musculoskeletal
injuries.
8 Musculoskeletal Trauma
210 of 17
Shock
3
Case Scenario
38-year-old female (102 kg), restrained driver,
high-speed, head-on collision with large truck
Prolonged extrication, marked deformity R
thigh, open fracture R lower leg.
Awake and alert, in severe pain; Vital
signs: HR 120; BP 90/50; RR 22; GCS 15
Two large-bore IVs in upper extremities,
resuscitation with isotonic fluids
M
I
S
T
8 Musculoskeletal Trauma
211 of 17
Shock
3
Case Scenario Progression
• Airway and breathing are OK
• Lower limb actively bleeding,
dressing applied
• Vital signs after fluids: HR 130;
BP 80/40; RR 24; GCS 14
• Chest x-ray, pelvic x-ray, and FAST
exam negative
8 Musculoskeletal Trauma
212 of 17
Shock
3
Discussion Questions:
1. What are the priorities for this
patient during the primary
survey and resuscitation?
2. How will you assess the injured
extremities at this point?
Case Details
• Airway and breathing are OK
• Lower limb bleeding, dressing
applied
• Vital signs after fluids: HR 130;
BP 80/40; RR 24; GCS 14
• Chest x-ray, pelvic x-ray, and
FAST exam negative
8 Musculoskeletal Trauma
213 of 17
Shock
3
Discussion Questions:
3. How much blood loss would you
expect from this patient’s
extremity injuries, and what is
the best way to control it?
4. How should femur and tibial
shaft fractures be stabilized?
Case Details
• Airway and breathing are OK
• Lower limb bleeding, dressing
applied
• Vital signs after fluids: HR 130;
BP 80/40; RR 24; GCS 14
• Chest x-ray, pelvic x-ray, and
FAST exam negative
8 Musculoskeletal Trauma
214 of 17
Shock
3
Case Scenario Progression
• Right dorsalis pedis and posterior tibial pulses absent
• Pulses remain absent after splinting.
• Blood soaking through dressings, R lower leg
• Dressing taken down, pulsatile bleeding noted
• Direct pressure applied to wound, followed by gauze packing
• Dressing quickly becomes saturated with blood again
8 Musculoskeletal Trauma
215 of 17
Shock
3
Discussion Question:
What is the next step in the
management of this patient’s
uncontrolled extremity
hemorrhage?
Case Details
• Right dorsalis pedis and posterior tibial
pulses absent
• Pulses remain absent after splinting.
• Blood soaking through dressings, R lower
leg
• Dressing taken down, pulsatile bleeding
noted
• Direct pressure applied to wound,
followed by gauze packing
• Dressing quickly becomes saturated with
blood again
8 Musculoskeletal Trauma
216 of 17
Shock
3
Case Scenario Progression
• Tourniquet applied and time of placement documented
• Bleeding controlled
• Distal pulses not palpable
8 Musculoskeletal Trauma
217 of 17
Shock
3
Discussion Question:
How would you manage the patient now?
• Tourniquet applied and
time of placement
documented
• Bleeding controlled
• Distal pulses not
palpable
Case Details
8 Musculoskeletal Trauma
218 of 17
Shock
3
Case Scenario Progression
• Patient’s hemodynamics improve with IV fluids
• Secondary survey begins
• Awaiting transfer due to no surgical capabilities at this facility
8 Musculoskeletal Trauma
219 of 17
Shock
3
Discussion Question:
1. How would you identify any limb-
threatening injuries or other
extremity injuries during the
secondary survey?
2. What x-rays should be ordered for this
patient prior to transfer?
• Patient’s hemodynamics
improve
• Secondary survey begins
• Awaiting transfer due to
no surgical capabilities at
this facility
Case Details
8 Musculoskeletal Trauma
220 of 17
Shock
3
Case Scenario Progression
• X-rays of the patient’s extremities are obtained.
• Radiographs do not cause a delay in transfer.
8 Musculoskeletal Trauma
221 of 17
Shock
3
Case Scenario Progression
• Vital signs: HR 105; BP 110/70; RR 24; GCS 15.
• Bleeding is controlled with tourniquet.
• Urgent transfer already initiated.
• No known drug allergies.
8 Musculoskeletal Trauma
222 of 17
Shock
3
Discussion Question:
1. What else should be done prior
to transfer
2. What antibiotics would you give
and at what dose?
3. How can you decrease the
patient’s discomfort and pain?
Case Details
• Vital signs: HR 105; BP 110/70; RR
24; GCS 15.
• Bleeding is controlled with
tourniquet.
• Your facility does not have the
resources to manage the patient’s
injuries, so urgent transfer
initiated.
8 Musculoskeletal Trauma
223 of 17
Shock
3
Case Scenario Progression
• The patient is transferred to the nearest trauma center, where a trauma
surgeon and an orthopedic surgeon are available to take the patient to
the operating room immediately
• Vital signs: HR 105; BP 110/70; RR 24; GCS 15
• Bleeding is controlled with tourniquet
• Patient received antibiotics prior to transfer
8 Musculoskeletal Trauma
224 of 17
Shock
3
Discussion Question:
1. What measures need to be
taken at the receiving hospital?
2. How can you make an early
diagnosis of compartment
syndrome?
3. What is the treatment for
compartment syndrome?
Case Details
• Patient is transferred to the nearest
trauma center, where a trauma surgeon
and an orthopedic surgeon are available
to take the patient to the operating room
immediately.
• Vital signs: HR 105; BP 110/70; RR 24;
GCS 15.
• Bleeding is controlled with tourniquet.
• Patient received antibiotics prior to
transfer.
8 Musculoskeletal Trauma
225 of 17
Shock
3
Case Scenario Conclusion
Patient taken to the operating room for urgent evaluation of
vascular injury, wound washout and external fixation of fracture.
8 Musculoskeletal Trauma
226 of 17
Shock
3
Any Questions?
8 Musculoskeletal Trauma
227 of 17
Shock
3
Review Objectives
By the end of this interactive discussion, you will be able to:
1. Explain the significance of musculoskeletal injuries in patients with multiple injuries.
2. Outline the priorities of the primary survey and resuscitation of patients with
extremity injuries.
3. Identify the adjuncts needed in the immediate treatment of life-threatening extremity
hemorrhage.
4. Describe key elements of the secondary survey of patients with musculoskeletal
trauma.
5. Explain the principles of the initial management of limb-threatening musculoskeletal
injuries.
8 Musculoskeletal Trauma
228 of 17
Shock
3
Key Learning Points
• Hemorrhage from long bone fractures can be significant
• Early splinting helps to control blood loss, reduce pain, and prevent further
neurovascular compromise and soft tissue injury
• Early weight-based dosing of antibiotics for patients with open fractures
• Compartment syndrome is a clinical diagnosis, and the treatment is
fasciotomy.
8 Musculoskeletal Trauma
Skill Station :
Adjuncts
Normal Chest X-ray
- Patient identity
- Density
- Centration
231 of 17
Shock
3 Chest X-ray
Massive left hematothorax
232 of 17
Shock
3 Chest X-ray
- Right pneumothorax
(collaps of right lung)
- Left pulmonary contusion
233 of 17
Shock
3 Chest X-ray
- Left pneumothorax
(chest tube inserted)
- Multiple rib fractures
- Left subcutaneous air
234 of 17
Shock
3
CXR
Chest X-ray
- Multiple rib fractures
(flail chest)
- Left subcutaneous air
- Fractures of :
-clavicle
-humeral neck
-scapula
Chest X-ray
- Abnormal gas left chest
(herniation of intraabdominal
organ, bowel)
- Diaphragm not visualized
(rupture of left diaphragma)
236 of 17
Shock
3 Chest X-ray
Widened mediastinum on CXR
237 of 17
Shock
3 Chest CT-scan
- Widened mediastinum
- Periaortic hematoma
(aortic injury)
238 of 17
Shock
3 Normal Pelvic X-ray
239 of 17
Shock
3 Pelvic X-ray
- Extensive widening of the
sacroiliac joints
- Pelvic diastasis
240 of 17
Shock
3 Pelvic X-ray
Bilateral inferior and superior
pelvic rami fractures
241 of 17
Shock
3 Pelvic Binder
before binder after binder
Foto : pelvic diasthesis
242 of 17
Shock
3 Pelvic X-ray
Unconscious and hypotensive
Foto : vertical shear fracture
1. vertical translation of the pelvis
and widened pubis
2. extensive widening of the
sacroiliac joints
3. inferior and superior pelvic rami
fractures
4. left acetabular fractures
Urethrogram
Foto :
No extravasation of contrast
material
244 of 17
Shock
3 Urethro-cyctogram
Clinical :
- Hypotension
- obvious unstable pelvis
- gross hematuria
- perineal ecchymosis
Foto :
No extravasation of contrast
material
245 of 17
Shock
3 Urethro-cyctogram
Clinical :
Gross hematuria
Foto :
Extravasation of contrast
material outsite bladder,
but confined to pelvis
(retroperitoneal bladder
rupture)
246 of 17
Shock
3 Urethro-cyctogram
Clinical :
- lower abdominal pain
- gross hematuria
Foto :
- diffuse contrast
extravasation
- not confined to the pelvis
(intraperitoneal bladder
rupture)
247 of 17
Shock
3
A Alignment and adequacy
A. Anterior vertebral line
B. Anterior spinal line
C. Posterior spinal line
D. Spinous process
Cervical Spine Anatomy
248 of 17
Shock
3
B Bone
C1 Atlas
C2 Odontoid
C3-C7
C2
C1
C7
Cervical Spine Anatomy
249 of 17
Shock
3
C Cartilage
D Dens
E Extra axial soft tissues
F Facets
Cervical Spine Anatomy
250 of 17
Shock
3 Pediatric Cervical Spine
normal subluxation of C2-C3
251 of 17
Shock
3
CT
Cervical Spine
normal
252 of 17
Shock
3 Cervical Spine
odontoid fracture, comminution and depression at the C1-C2 interfacet joint
Open Mouth and Lateral View
C-1 burst (Jefferson fracture)
254 of 17
Shock
3 Cervical Spine
complete subluxation of C7-T1
subluxation of C6-C7
255 of 17
Shock
3 Thoraco-lumbal Spine
fracture dislocation of T3-T4 fracture of L2 fracture of L3
256 of 17
Shock
3
257 of 17
Shock
3
The most significant difference between burns and other injuries is
that the consequences of burn injury are directly linked to the extent
of the inflammatory response to the injury.
9 Thermal Injuries
258 of 17
Shock
3
Objectives
By the end of this interactive discussion, you will be able to:
1. Discuss the potential risks to the airway of patients with burn injuries.
2. Discuss resuscitation strategies for patients with burns.
3. Estimate the extent of a simulated patient’s burn injury.
4. Describe the appropriate management of burn injuries, including circumferential
burns.
5. Discuss the proper handover method for patients with burns.
6. Describe management of patients with hypothermia, including rewarming risks.
7. Describe the tissue effects of cold injury.
8. Describe the initial treatment of patients with tissue injury from cold exposure.
9 Thermal Injuries
259 of 17
Shock
3
Case Scenario
29-year-old male jumps from first story of a
burning house; clothes on fire; bystanders
extinguish fire and summon EMS
Conscious, agitated, voice normal, complaining of
abdominal and leg pain; head and upper body
extensively burned
None reported
None reported
M
I
S
T
9 Thermal Injuries
260 of 17
Shock
3
Discussion Questions:
1. What are your immediate
priorities?
2. Is this patient’s airway at risk?
Why, or why not?
3. What are some physical
examination findings that
suggest the airway is injured?
Case Details
M
29-year-old male jumps from first story
of a burning house; clothes on fire;
bystanders extinguish fire and summon
EMS
Conscious, agitated, voice normal,
complaining of abdominal and leg pain;
head and upper body extensively burned
None reported
None reported
I
S
T
9 Thermal Injuries
261 of 17
Shock
3
Discussion Questions:
4. How can you decrease injury
progression from the burn?
5. What are ways to control this
patient’s pain?
Case Details
M
29-year-old male jumps from first story
of a burning house; clothes on fire;
bystanders extinguish fire and summon
EMS
Conscious, agitated, voice normal,
complaining of abdominal and leg pain;
head and upper body extensively burned
None reported
None reported
I
S
T
9 Thermal Injuries
262 of 17
Shock
3
Discussion Questions:
6. What other potential injuries do
you need to consider in this
patient?
7. What other thermal injury
consequences do you need to
consider?
Case Details
M
29-year-old male jumps from first story
of a burning house; clothes on fire;
bystanders extinguish fire and summon
EMS
Conscious, agitated, voice normal,
complaining of abdominal and leg pain;
head and upper body extensively burned
None reported
None reported
I
S
T
9 Thermal Injuries
263 of 17
Shock
3
Case Scenario Progression
• Patient is intubated
• IV access obtained in the antecubital fossa through burned skin
• Patient exposed fully: face, anterior torso, and bilateral lower
extremities nearly completely burned circumferentially
• Foley catheter placed, minimal dark urine
9 Thermal Injuries
264 of 17
Shock
3
Discussion Question:
1. What is estimated size of the
burn in this patient?
9 Thermal Injuries
265 of 17
Shock
3
Discussion Questions:
2. What is the significance of the
dark urine?
Case Details
• Patient is intubated
• IV access obtained in the antecubital
fossa through burned skin
• Patient exposed fully: face, anterior
torso, and bilateral lower extremities
nearly completely burned
circumferentially
• Foley catheter placed, minimal dark urine
9 Thermal Injuries
266 of 17
Shock
3
Case Scenario Progression
• Initial ABCDEs addressed
• Estimated TBSA burned calculated
• Second- and third-degree burn wounds throughout
9 Thermal Injuries
267 of 17
Shock
3
Discussion Question:
How will you manage this patient’s
burns to prevent further morbidity
and mortality?
• Initial ABCDEs addressed
• Estimated TBSA burned
calculated
• Second- and third-
degree burn wounds
throughout
Case Details
9 Thermal Injuries
268 of 17
Shock
3
Case Scenario Progression
• Fluid resuscitation begun
• Urine output < 30 mL/hr
• Fluid rate increased
• Chest and extremity x-rays ordered
9 Thermal Injuries
269 of 17
Shock
3
Discussion Question:
How do resuscitation strategies differ
in burn resuscitation from other types
of trauma resuscitation?
• Fluid resuscitation begun
• Urine output < 30 mL/hr
• Fluid rate increased
• Chest and extremity x-
rays ordered
Case Details
9 Thermal Injuries
9 Thermal Injuries
270 of 17
Shock
3
Case Scenario Progression
• Secondary survey: calves of both legs firm and edematous
9 Thermal Injuries
271 of 17
Shock
3
Discussion Question:
What is the significance of finding
swelling and firm compartments on
the secondary survey?
• Calves of both legs firm and
edematous
Case Details
9 Thermal Injuries
272 of 17
Shock
3
Case Scenario Conclusion
• Transfer arrangements made
• Decision to perform escharotomies prior to transfer
9 Thermal Injuries
273 of 17
Shock
3
Discussion Question:
As the team prepares for transfer and
to carry out the hand-off to the
receiving facility, what are some
important elements that should be
communicated, and where should
they be documented?
• Transfer arrangements
made
• Decision to perform
escharotomies prior to
transfer
Case Details
9 Thermal Injuries
274 of 17
Shock
3
Case Scenario #2
35-year-old female brought to hospital after
being lost for two days, snowmobiling in -30° C
(-22° F) weather
None reported
Core body temperature 30° C (86° F)
None reported
M
I
S
T
9 Thermal Injuries
275 of 17
Shock
3
Discussion Questions:
1. How would you initially treat
this patient?
2. How and why would you
monitor this patient during the
rewarming?
Case Details
M
35-year-old female brought to
hospital after being lost for two
days, snowmobiling in -30° C
weather
None reported
Core body temperature 30° C
None reported
I
S
T
9 Thermal Injuries
276 of 17
Shock
3
Any Questions?
9 Thermal Injuries
277 of 17
Shock
3
Objectives
By the end of this interactive discussion, you will be able to:
1. Discuss the potential risks to the airway of patients with burn injuries.
2. Discuss resuscitation strategies for patients with burns.
3. Estimate the extent of a simulated patient’s burn injury.
4. Describe the appropriate management of burn injuries, including circumferential
burns.
5. Discuss the proper handover method for patients with burns.
6. Describe management of patients with hypothermia, including rewarming risks.
7. Describe the tissue effects of cold injury.
8. Describe the initial treatment of patients with tissue injury from cold exposure.
9 Thermal Injuries
278 of 17
Shock
3
Key Learning Points
1. The most significant difference between burns and other injuries is that the
consequences of burn injury are directly linked to the extent of the inflammatory
response to the injury. This drives the rate and amount of edema formation.
2. The airway can become obstructed not only from direct injury (e.g., inhalation injury),
but also from the massive edema resulting from the burn injury. Edema is typically not
present immediately, and signs of obstruction may initially be subtle until the patient is
in crisis.
3. In contrast to resuscitation for other types of trauma in which fluid deficit is typically
secondary to hemorrhagic losses, burn resuscitation is required to replace the ongoing
losses from capillary leak due to inflammation.
9 Thermal Injuries
279 of 17
Shock
3
Key Learning Points
4. A fresh burn is a clean area that must be protected from contamination.
5. Ensure that there are flow sheets documenting the patient history, injury, IV fluids given,
and urinary output. The flow sheet should be sent with the patient on transfer.
6. Although rapid rewarming is essential for management of frostbite and hypothermia,
reperfusion can cause physiologic changes that need to be managed.
9 Thermal Injuries
10 Pediatric Trauma Tenth Edition
281 of 17
Shock
3
Injury remains the most common cause of death and disability in
childhood. Injury morbidity and mortality surpass all major diseases in
children and young adults, making trauma the most serious public
health and health care problem in this population.
10 Pediatric Trauma
282 of 17
Shock
3
Objectives
By the end of this interactive discussion, you will be able to:
1. Identify the initial priorities of trauma assessment and management for children.
2. Describe the most appropriate interventions for managing difficult airways in pediatric
trauma patients.
3. Recognize the most common causes of cardiac arrest in children.
4. Identify methods for obtaining venous access in children.
5. Discuss how to determine drug and fluid dosages in children.
6. Evaluate for nonaccidental trauma in a pediatric trauma case.
10 Pediatric Trauma
283 of 17
Shock
3
Case Scenario
3-year-old boy falls 10 meters (32 feet) out of an
apartment window onto pavement
Patient does not open eyes, moans incomprehensibly,
extends abnormally when stimulated
Unresponsive on arrival to ED at a small rural hospital,
pupils unequal, blood coming from R ear, breathing
rapidly, pale, mottled extremities Vital signs: BP 74/57;
HR 156; RR 49
None reported
M
I
S
T
10 Pediatric Trauma
284 of 17
Shock
3
Discussion Question:
What are the priorities in
evaluating a small child with
multisystem trauma?
Case Details
M
3-year-old boy falls 10 meters (32 ft) out
of an apartment window onto pavement
Patient does not open eyes, moans
incomprehensibly, extends abnormally
when stimulated
Unresponsive on arrival to ED at a small
rural hospital, pupils unequal, blood
coming from R ear, breathing rapidly, pale,
mottled extremities Vital signs: BP 74/57;
HR 156; RR 49
None reported
I
S
T
10 Pediatric Trauma
285 of 17
Shock
3
Case Scenario Progression
• Vital signs: BP 74/57; HR 156; RR 49; O2 sat 85%
• Breath sounds symmetrical
10 Pediatric Trauma
286 of 17
Shock
3
Discussion Question:
1. What is the most common cause of cardiac arrest in
children?
10 Pediatric Trauma
287 of 17
Shock
3
Discussion Questions:
2. What steps and maneuvers
would you use to manage this
patient’s airway?
3. Based on the information given,
is this child in shock? If so, what
type of shock is exhibited?
Case Details
• Vital signs: BP 74/57; HR 156; RR
49; O2 sat 85%
• Breath sounds symmetrical
10 Pediatric Trauma
288 of 17
Shock
3
Case Scenario Progression
• Two failed attempts at intubation
• Attempt to position laryngeal mask unsuccessful
10 Pediatric Trauma
289 of 17
Shock
3
Discussion Question:
What would your next step be in securing an airway for this
patient?
10 Pediatric Trauma
290 of 17
Shock
3
Case Scenario Progression
• Needle cricothryroidotomy performed, airway secured
• Vital signs: BP 74/57; HR 156; RR 49
• Several failed attempts at peripheral IV access in both antecubital
fossae
10 Pediatric Trauma
291 of 17
Shock
3
Discussion Question:
1. How would you secure vascular
access to give fluids?
2. How would you manage this
patient’s hypotension and
determine the appropriate fluid
regime?
• Needle cricothryroidotomy
performed, airway secured
• Vital signs: BP 74/57; HR
156; RR 49
• Several failed attempts at
peripheral IV access in both
antecubital fossae
Case Details
10 Pediatric Trauma
292 of 17
Shock
3
Case Scenario Progression
• Isotonic crystalloid and O-negative blood given with good response
• Vital signs: HR 110; BP 90/60
• Chest x-ray: pulmonary contusions
10 Pediatric Trauma
293 of 17
Shock
3
Discussion Question:
Why do children commonly develop pulmonary contusions following
trauma, even in the absence of rib fractures?
10 Pediatric Trauma
294 of 17
Shock
3
Case Scenario Progression
• Pelvic x-ray: normal
• Inconsistencies noted in family’s explanation of fall
• Skeletal survey: healed transphyseal distal L humerus fracture; healing
classic L femur metaphyseal lesion, with periostitis extending into the
femoral diaphysis
10 Pediatric Trauma
295 of 17
Shock
3
Discussion Questions:
1. Do the x-ray findings raise the
suspicion of child maltreatment?
If so, why?
2. What are the appropriate steps
to take if child maltreatment is
suspected?
• Pelvic x-ray: normal
• Inconsistencies noted in family’s
explanation of fall
• Skeletal survey: healed
transphyseal distal L humerus
fracture; healing classic L femur
metaphyseal lesion, with
periostitis extending into the
femoral diaphysis
Case Details
10 Pediatric Trauma
296 of 17
Shock
3
Case Scenario Conclusion
• Transfer to nearest pediatric trauma center after intubation and normalization of
hemodynamics
• CT of head and abdomen at receiving facility: intracerebral contusions, moderate
splenic contusion, and subscapular hematoma
• Managed in ICU until gas exchange normalized, extubated
• Splenic injury does not require surgery
• Case reported to the appropriate social and legal services
• Outpatient brain injury follow up for 6 months
10 Pediatric Trauma
297 of 17
Shock
3
Any Questions?
10 Pediatric Trauma
298 of 17
Shock
3
Review Objectives
By the end of this interactive discussion, you will be able to:
1. Identify the initial priorities of trauma assessment and management for children.
2. Describe the most appropriate interventions for managing difficult airways in pediatric
trauma patients.
3. Recognize the most common causes of cardiac arrest in children.
4. Identify methods for obtaining venous access in children.
5. Discuss how to determine drug and fluid dosages in children.
6. Evaluate for nonaccidental trauma in a pediatric trauma case.
10 Pediatric Trauma
299 of 17
Shock
3
Key Learning Points
1. The initial priorities of trauma assessment and management are the same for children
and adults.
2. Surgical cricothyroidotomy is generally considered to be unsafe in small children (<12)
due to the small size of the cricothyroid membrane and proximity to vocal cords. Needle
cricothyroidotomy is preferred as a temporizing solution until other preparations are
made.
3. Hypoxia and respiratory compromise are the most common causes of cardiac arrest in
children.
4. Emergent venous access in children can be difficult. If unable to obtain peripheral
access, intraosseous access should be obtained immediately.
10 Pediatric Trauma
300 of 17
Shock
3
Key Learning Points
5. Determination of weight is essential to pediatric trauma care in order to dose drugs and
guide fluid and blood resuscitation. This can be accomplished by history, length-based
resuscitation tape, or specialty stretchers with integrated scales.
6. Blunt solid organ injury in pediatric patients is usually managed non-operatively by a
surgeon unless the patient is hemodynamically unstable or there are other indications
for surgery.
7. Non-accidental trauma is a significant source of injury in children and has a higher
mortality rate than corresponding accidental injuries. Specific injury patterns exist that
should heighten concerns. Clinicians must have a high index of suspicion and report
these cases appropriately.
10 Pediatric Trauma
10 Pediatric Trauma Tenth Edition
302 of 17
Shock
3
When managing geriatric patients with trauma, the effects of aging on
physiological function and the impact of preexisting conditions and
medications cannot be overemphasized.
11 Geriatric Trauma
303 of 17
Shock
3
Objectives
By the end of this interactive discussion, you will be able to:
1. Describe common mechanisms of injury seen in older adults.
2. Apply the ATLS principles to the management of an elderly trauma
patient.
3. Understand the physiologic changes that occur with aging and how they
affect the geriatric patient’s injury and response to trauma.
4. Understand the common signs and causes of elder maltreatment.
11 Geriatric Trauma
304 of 17
Shock
3
Case Scenario
82-year-old female fell down 6 stairs;
husband called ambulance,
transported to your rural ED
None reported
Vital signs: RR 22; HR 64; BP 160/80;
GCS 13
None reported
M
I
S
T
11 Geriatric Trauma
305 of 17
Shock
3
Discussion Questions:
1. What injuries would you suspect
in this elderly patient?
2. What unique elements of the
AMPLE history should you be
alert to?
3. What are the key aspects of the
initial assessment and
management of this patient?
Case Details
M
82-year-old female fell down 6
stairs; husband called
ambulance, transported to your
rural ED
None reported
Vital signs: RR 22; HR 64; BP
160/80; GCS 13
None reported
I
S
T
11 Geriatric Trauma
306 of 17
Shock
3
Case Scenario Progression
• Examination
• Vital signs unchanged on ED arrival
• Patient confused, localizes to pressure
• Multiple bruises
• Less movement upper compared with lower extremities
• AMPLE: taking a direct thrombin inhibitor, beta blocker for hypertension
• Investigations:
• Chest x-ray: multiple rib fractures
• Head CT: subdural hematoma with shift, few small intracerebral contusions
• Pelvic x-ray: pubic rami fractures
11 Geriatric Trauma
307 of 17
Shock
3
Discussion Questions:
1. What challenges and pitfalls
might you encounter if you
need to intubate the patient?
2. With respect to her rib
fractures, why is this patient at
higher risk for mortality than a
younger individual?
Case Details
• Vital signs unchanged on ED arrival
• Patient confused, localizes to pressure
• Multiple bruises
• Less movement upper compared with lower
extremities
• AMPLE: taking a direct thrombin inhibitor,
beta blocker for hypertension
• Chest x-ray: multiple rib fractures
• Head CT: subdural hematoma with shift, few
small intracerebral contusions
• Pelvic x-ray: pubic rami fractures
11 Geriatric Trauma
308 of 17
Shock
3
Case Scenario Progression
• Transfer not possible due to severe weather
• Admitted to ICU for monitoring
• Rib fracture pain controlled with narcotics
• Respiratory status and level of consciousness continue to decline
• Repeat head CT: progression of subdural hematoma
• Neurosurgeon by telemedicine recommends craniotomy for
evacuation of the hematoma when transfer is available
11 Geriatric Trauma
309 of 17
Shock
3
Discussion Questions:
1. What are the potential causes
of declining mental status in
this patient? What are the most
likely?
2. What is the impact of a
prolonged ICU admission and
hospital stay on this elderly
patient’s likelihood of returning
to independent living?
Case Details
• Transfer not possible due to severe weather
• Admitted to ICU for monitoring
• Rib fracture pain controlled with narcotics
• Respiratory status and level of consciousness
continue to decline
• Repeat head CT: progression of subdural
hematoma
• Neurosurgeon by telemedicine recommends
craniotomy for evacuation of the hematoma
when transfer is available
11 Geriatric Trauma
310 of 17
Shock
3
Discussion Questions:
3. Does the patient’s injury pattern
raise suspicion about elder
maltreatment?
4. What type of injuries would
raise your suspicion about elder
maltreatment?
Case Details
M
82-year-old female fell down 6
stairs; husband called
ambulance, transported to your
rural ED
None reported
Vital signs: RR 22; HR 64; BP
160/80; GCS 13
None reported
I
S
T
11 Geriatric Trauma
311 of 17
Shock
3
Case Scenario Conclusion
• Husband states that the patient would not want intubation nor life-
sustaining interventions if there is no chance of living independently
again.
• He provides advance directive and living will documenting her wishes
and his status as health care power of attorney.
11 Geriatric Trauma
312 of 17
Shock
3
Any Questions?
11 Geriatric Trauma
PPT Advance Trauma Life Support edisi 10
PPT Advance Trauma Life Support edisi 10
PPT Advance Trauma Life Support edisi 10
PPT Advance Trauma Life Support edisi 10
PPT Advance Trauma Life Support edisi 10
PPT Advance Trauma Life Support edisi 10
PPT Advance Trauma Life Support edisi 10
PPT Advance Trauma Life Support edisi 10
PPT Advance Trauma Life Support edisi 10
PPT Advance Trauma Life Support edisi 10
PPT Advance Trauma Life Support edisi 10
PPT Advance Trauma Life Support edisi 10
PPT Advance Trauma Life Support edisi 10
PPT Advance Trauma Life Support edisi 10
PPT Advance Trauma Life Support edisi 10
PPT Advance Trauma Life Support edisi 10
PPT Advance Trauma Life Support edisi 10
PPT Advance Trauma Life Support edisi 10
PPT Advance Trauma Life Support edisi 10
PPT Advance Trauma Life Support edisi 10
PPT Advance Trauma Life Support edisi 10
PPT Advance Trauma Life Support edisi 10
PPT Advance Trauma Life Support edisi 10
PPT Advance Trauma Life Support edisi 10
PPT Advance Trauma Life Support edisi 10
PPT Advance Trauma Life Support edisi 10
PPT Advance Trauma Life Support edisi 10
PPT Advance Trauma Life Support edisi 10
PPT Advance Trauma Life Support edisi 10
PPT Advance Trauma Life Support edisi 10
PPT Advance Trauma Life Support edisi 10
PPT Advance Trauma Life Support edisi 10
PPT Advance Trauma Life Support edisi 10
PPT Advance Trauma Life Support edisi 10
PPT Advance Trauma Life Support edisi 10
PPT Advance Trauma Life Support edisi 10
PPT Advance Trauma Life Support edisi 10
PPT Advance Trauma Life Support edisi 10
PPT Advance Trauma Life Support edisi 10
PPT Advance Trauma Life Support edisi 10
PPT Advance Trauma Life Support edisi 10

More Related Content

Similar to PPT Advance Trauma Life Support edisi 10

Emergency medicine:The most wanted medical speciality in India
Emergency medicine:The most wanted medical  speciality in India Emergency medicine:The most wanted medical  speciality in India
Emergency medicine:The most wanted medical speciality in India
Dr.Venugopalan Poovathum Parambil
 
Casualty, emergency and trauma A to Z
Casualty, emergency and trauma A to ZCasualty, emergency and trauma A to Z
Casualty, emergency and trauma A to Z
dr tushar chokshi
 
87 muster2014 Nontsikelelo
87 muster2014 Nontsikelelo87 muster2014 Nontsikelelo
87 muster2014 NontsikeleloMuster2014
 
4 5walls-johnson-goodman16
4 5walls-johnson-goodman164 5walls-johnson-goodman16
4 5walls-johnson-goodman16
afacct
 
DNB EM :Good academics in emergency training progam
DNB EM :Good academics in emergency training progamDNB EM :Good academics in emergency training progam
DNB EM :Good academics in emergency training progam
Dr.Venugopalan Poovathum Parambil
 
Facilitation of clinical reasoning during bedside teaching workshop for clini...
Facilitation of clinical reasoning during bedside teaching workshop for clini...Facilitation of clinical reasoning during bedside teaching workshop for clini...
Facilitation of clinical reasoning during bedside teaching workshop for clini...
Muhammed Elhady Muhammed Elgasim
 
Paramedic update
Paramedic updateParamedic update
Paramedic update
emscaptain
 
Test Bank for Brunner & Suddarth's Textbook of medical surgical nursing 14th ...
Test Bank for Brunner & Suddarth's Textbook of medical surgical nursing 14th ...Test Bank for Brunner & Suddarth's Textbook of medical surgical nursing 14th ...
Test Bank for Brunner & Suddarth's Textbook of medical surgical nursing 14th ...
nursing premium
 
Evidence Based Medicine Master degree Course (Lecture 1): Formulation of clin...
Evidence Based Medicine Master degree Course (Lecture 1): Formulation of clin...Evidence Based Medicine Master degree Course (Lecture 1): Formulation of clin...
Evidence Based Medicine Master degree Course (Lecture 1): Formulation of clin...
Iman El Sayed
 
Improvement U Adult Mock Code Presentation
Improvement U Adult Mock Code PresentationImprovement U Adult Mock Code Presentation
Improvement U Adult Mock Code PresentationKim Nelson
 
Paramedic update part 1
Paramedic update part 1Paramedic update part 1
Paramedic update part 1
emscaptain
 
Trauma surgery: a lecturer for FY programme doctors. Lunevicius R. 2017
Trauma surgery: a lecturer for FY programme doctors. Lunevicius R. 2017Trauma surgery: a lecturer for FY programme doctors. Lunevicius R. 2017
Trauma surgery: a lecturer for FY programme doctors. Lunevicius R. 2017
Raimundas Lunevicius
 
GEMC - Trauma Patient Care in the Emergency Department : Pitfalls to Avoid
GEMC - Trauma Patient Care in the Emergency Department : Pitfalls to AvoidGEMC - Trauma Patient Care in the Emergency Department : Pitfalls to Avoid
GEMC - Trauma Patient Care in the Emergency Department : Pitfalls to Avoid
Open.Michigan
 
Ebm Nahid Sherbini
Ebm Nahid SherbiniEbm Nahid Sherbini
Ebm Nahid Sherbini
Nahid Sherbini
 
MEU WORKSHOP Clinical Skills Education
MEU WORKSHOP Clinical Skills EducationMEU WORKSHOP Clinical Skills Education
MEU WORKSHOP Clinical Skills Education
Devan Pannen
 
Journal club presentation (chest tube) _.pptx dr shakil
Journal club presentation (chest tube) _.pptx dr shakilJournal club presentation (chest tube) _.pptx dr shakil
Journal club presentation (chest tube) _.pptx dr shakil
Shakil Ahmad
 
.PCI.pdf
.PCI.pdf.PCI.pdf
.PCI.pdf
ssuser1353e5
 
Implementing psychosocial care into routine practice: making it easy
Implementing psychosocial care into routine practice: making it easyImplementing psychosocial care into routine practice: making it easy
Implementing psychosocial care into routine practice: making it easy
Cancer Institute NSW
 
TEST BANK For Critical Thinking, Clinical Reasoning, and Clinical Judgment A ...
TEST BANK For Critical Thinking, Clinical Reasoning, and Clinical Judgment A ...TEST BANK For Critical Thinking, Clinical Reasoning, and Clinical Judgment A ...
TEST BANK For Critical Thinking, Clinical Reasoning, and Clinical Judgment A ...
robinsonayot
 
Evidence based medicine
Evidence based medicineEvidence based medicine
Evidence based medicine
Dr Vaziri
 

Similar to PPT Advance Trauma Life Support edisi 10 (20)

Emergency medicine:The most wanted medical speciality in India
Emergency medicine:The most wanted medical  speciality in India Emergency medicine:The most wanted medical  speciality in India
Emergency medicine:The most wanted medical speciality in India
 
Casualty, emergency and trauma A to Z
Casualty, emergency and trauma A to ZCasualty, emergency and trauma A to Z
Casualty, emergency and trauma A to Z
 
87 muster2014 Nontsikelelo
87 muster2014 Nontsikelelo87 muster2014 Nontsikelelo
87 muster2014 Nontsikelelo
 
4 5walls-johnson-goodman16
4 5walls-johnson-goodman164 5walls-johnson-goodman16
4 5walls-johnson-goodman16
 
DNB EM :Good academics in emergency training progam
DNB EM :Good academics in emergency training progamDNB EM :Good academics in emergency training progam
DNB EM :Good academics in emergency training progam
 
Facilitation of clinical reasoning during bedside teaching workshop for clini...
Facilitation of clinical reasoning during bedside teaching workshop for clini...Facilitation of clinical reasoning during bedside teaching workshop for clini...
Facilitation of clinical reasoning during bedside teaching workshop for clini...
 
Paramedic update
Paramedic updateParamedic update
Paramedic update
 
Test Bank for Brunner & Suddarth's Textbook of medical surgical nursing 14th ...
Test Bank for Brunner & Suddarth's Textbook of medical surgical nursing 14th ...Test Bank for Brunner & Suddarth's Textbook of medical surgical nursing 14th ...
Test Bank for Brunner & Suddarth's Textbook of medical surgical nursing 14th ...
 
Evidence Based Medicine Master degree Course (Lecture 1): Formulation of clin...
Evidence Based Medicine Master degree Course (Lecture 1): Formulation of clin...Evidence Based Medicine Master degree Course (Lecture 1): Formulation of clin...
Evidence Based Medicine Master degree Course (Lecture 1): Formulation of clin...
 
Improvement U Adult Mock Code Presentation
Improvement U Adult Mock Code PresentationImprovement U Adult Mock Code Presentation
Improvement U Adult Mock Code Presentation
 
Paramedic update part 1
Paramedic update part 1Paramedic update part 1
Paramedic update part 1
 
Trauma surgery: a lecturer for FY programme doctors. Lunevicius R. 2017
Trauma surgery: a lecturer for FY programme doctors. Lunevicius R. 2017Trauma surgery: a lecturer for FY programme doctors. Lunevicius R. 2017
Trauma surgery: a lecturer for FY programme doctors. Lunevicius R. 2017
 
GEMC - Trauma Patient Care in the Emergency Department : Pitfalls to Avoid
GEMC - Trauma Patient Care in the Emergency Department : Pitfalls to AvoidGEMC - Trauma Patient Care in the Emergency Department : Pitfalls to Avoid
GEMC - Trauma Patient Care in the Emergency Department : Pitfalls to Avoid
 
Ebm Nahid Sherbini
Ebm Nahid SherbiniEbm Nahid Sherbini
Ebm Nahid Sherbini
 
MEU WORKSHOP Clinical Skills Education
MEU WORKSHOP Clinical Skills EducationMEU WORKSHOP Clinical Skills Education
MEU WORKSHOP Clinical Skills Education
 
Journal club presentation (chest tube) _.pptx dr shakil
Journal club presentation (chest tube) _.pptx dr shakilJournal club presentation (chest tube) _.pptx dr shakil
Journal club presentation (chest tube) _.pptx dr shakil
 
.PCI.pdf
.PCI.pdf.PCI.pdf
.PCI.pdf
 
Implementing psychosocial care into routine practice: making it easy
Implementing psychosocial care into routine practice: making it easyImplementing psychosocial care into routine practice: making it easy
Implementing psychosocial care into routine practice: making it easy
 
TEST BANK For Critical Thinking, Clinical Reasoning, and Clinical Judgment A ...
TEST BANK For Critical Thinking, Clinical Reasoning, and Clinical Judgment A ...TEST BANK For Critical Thinking, Clinical Reasoning, and Clinical Judgment A ...
TEST BANK For Critical Thinking, Clinical Reasoning, and Clinical Judgment A ...
 
Evidence based medicine
Evidence based medicineEvidence based medicine
Evidence based medicine
 

Recently uploaded

Pulmonary Thromboembolism - etilogy, types, medical- Surgical and nursing man...
Pulmonary Thromboembolism - etilogy, types, medical- Surgical and nursing man...Pulmonary Thromboembolism - etilogy, types, medical- Surgical and nursing man...
Pulmonary Thromboembolism - etilogy, types, medical- Surgical and nursing man...
VarunMahajani
 
New Directions in Targeted Therapeutic Approaches for Older Adults With Mantl...
New Directions in Targeted Therapeutic Approaches for Older Adults With Mantl...New Directions in Targeted Therapeutic Approaches for Older Adults With Mantl...
New Directions in Targeted Therapeutic Approaches for Older Adults With Mantl...
i3 Health
 
Lung Cancer: Artificial Intelligence, Synergetics, Complex System Analysis, S...
Lung Cancer: Artificial Intelligence, Synergetics, Complex System Analysis, S...Lung Cancer: Artificial Intelligence, Synergetics, Complex System Analysis, S...
Lung Cancer: Artificial Intelligence, Synergetics, Complex System Analysis, S...
Oleg Kshivets
 
Charaka Samhita Sutra Sthana 9 Chapter khuddakachatuspadadhyaya
Charaka Samhita Sutra Sthana 9 Chapter khuddakachatuspadadhyayaCharaka Samhita Sutra Sthana 9 Chapter khuddakachatuspadadhyaya
Charaka Samhita Sutra Sthana 9 Chapter khuddakachatuspadadhyaya
Dr KHALID B.M
 
ANATOMY AND PHYSIOLOGY OF URINARY SYSTEM.pptx
ANATOMY AND PHYSIOLOGY OF URINARY SYSTEM.pptxANATOMY AND PHYSIOLOGY OF URINARY SYSTEM.pptx
ANATOMY AND PHYSIOLOGY OF URINARY SYSTEM.pptx
Swetaba Besh
 
BRACHYTHERAPY OVERVIEW AND APPLICATORS
BRACHYTHERAPY OVERVIEW  AND  APPLICATORSBRACHYTHERAPY OVERVIEW  AND  APPLICATORS
BRACHYTHERAPY OVERVIEW AND APPLICATORS
Krishan Murari
 
Surat @ℂall @Girls ꧁❤8527049040❤꧂@ℂall @Girls Service Vip Top Model Safe
Surat @ℂall @Girls ꧁❤8527049040❤꧂@ℂall @Girls Service Vip Top Model SafeSurat @ℂall @Girls ꧁❤8527049040❤꧂@ℂall @Girls Service Vip Top Model Safe
Surat @ℂall @Girls ꧁❤8527049040❤꧂@ℂall @Girls Service Vip Top Model Safe
Savita Shen $i11
 
HOT NEW PRODUCT! BIG SALES FAST SHIPPING NOW FROM CHINA!! EU KU DB BK substit...
HOT NEW PRODUCT! BIG SALES FAST SHIPPING NOW FROM CHINA!! EU KU DB BK substit...HOT NEW PRODUCT! BIG SALES FAST SHIPPING NOW FROM CHINA!! EU KU DB BK substit...
HOT NEW PRODUCT! BIG SALES FAST SHIPPING NOW FROM CHINA!! EU KU DB BK substit...
GL Anaacs
 
Superficial & Deep Fascia of the NECK.pptx
Superficial & Deep Fascia of the NECK.pptxSuperficial & Deep Fascia of the NECK.pptx
Superficial & Deep Fascia of the NECK.pptx
Dr. Rabia Inam Gandapore
 
Prix Galien International 2024 Forum Program
Prix Galien International 2024 Forum ProgramPrix Galien International 2024 Forum Program
Prix Galien International 2024 Forum Program
Levi Shapiro
 
THOA 2.ppt Human Organ Transplantation Act
THOA 2.ppt Human Organ Transplantation ActTHOA 2.ppt Human Organ Transplantation Act
THOA 2.ppt Human Organ Transplantation Act
DrSathishMS1
 
Physiology of Special Chemical Sensation of Taste
Physiology of Special Chemical Sensation of TastePhysiology of Special Chemical Sensation of Taste
Physiology of Special Chemical Sensation of Taste
MedicoseAcademics
 
MANAGEMENT OF ATRIOVENTRICULAR CONDUCTION BLOCK.pdf
MANAGEMENT OF ATRIOVENTRICULAR CONDUCTION BLOCK.pdfMANAGEMENT OF ATRIOVENTRICULAR CONDUCTION BLOCK.pdf
MANAGEMENT OF ATRIOVENTRICULAR CONDUCTION BLOCK.pdf
Jim Jacob Roy
 
24 Upakrama.pptx class ppt useful in all
24 Upakrama.pptx class ppt useful in all24 Upakrama.pptx class ppt useful in all
24 Upakrama.pptx class ppt useful in all
DrSathishMS1
 
Charaka Samhita Sutra sthana Chapter 15 Upakalpaniyaadhyaya
Charaka Samhita Sutra sthana Chapter 15 UpakalpaniyaadhyayaCharaka Samhita Sutra sthana Chapter 15 Upakalpaniyaadhyaya
Charaka Samhita Sutra sthana Chapter 15 Upakalpaniyaadhyaya
Dr KHALID B.M
 
Evaluation of antidepressant activity of clitoris ternatea in animals
Evaluation of antidepressant activity of clitoris ternatea in animalsEvaluation of antidepressant activity of clitoris ternatea in animals
Evaluation of antidepressant activity of clitoris ternatea in animals
Shweta
 
heat stroke and heat exhaustion in children
heat stroke and heat exhaustion in childrenheat stroke and heat exhaustion in children
heat stroke and heat exhaustion in children
SumeraAhmad5
 
Maxilla, Mandible & Hyoid Bone & Clinical Correlations by Dr. RIG.pptx
Maxilla, Mandible & Hyoid Bone & Clinical Correlations by Dr. RIG.pptxMaxilla, Mandible & Hyoid Bone & Clinical Correlations by Dr. RIG.pptx
Maxilla, Mandible & Hyoid Bone & Clinical Correlations by Dr. RIG.pptx
Dr. Rabia Inam Gandapore
 
Surgical Site Infections, pathophysiology, and prevention.pptx
Surgical Site Infections, pathophysiology, and prevention.pptxSurgical Site Infections, pathophysiology, and prevention.pptx
Surgical Site Infections, pathophysiology, and prevention.pptx
jval Landero
 
Hemodialysis: Chapter 3, Dialysis Water Unit - Dr.Gawad
Hemodialysis: Chapter 3, Dialysis Water Unit - Dr.GawadHemodialysis: Chapter 3, Dialysis Water Unit - Dr.Gawad
Hemodialysis: Chapter 3, Dialysis Water Unit - Dr.Gawad
NephroTube - Dr.Gawad
 

Recently uploaded (20)

Pulmonary Thromboembolism - etilogy, types, medical- Surgical and nursing man...
Pulmonary Thromboembolism - etilogy, types, medical- Surgical and nursing man...Pulmonary Thromboembolism - etilogy, types, medical- Surgical and nursing man...
Pulmonary Thromboembolism - etilogy, types, medical- Surgical and nursing man...
 
New Directions in Targeted Therapeutic Approaches for Older Adults With Mantl...
New Directions in Targeted Therapeutic Approaches for Older Adults With Mantl...New Directions in Targeted Therapeutic Approaches for Older Adults With Mantl...
New Directions in Targeted Therapeutic Approaches for Older Adults With Mantl...
 
Lung Cancer: Artificial Intelligence, Synergetics, Complex System Analysis, S...
Lung Cancer: Artificial Intelligence, Synergetics, Complex System Analysis, S...Lung Cancer: Artificial Intelligence, Synergetics, Complex System Analysis, S...
Lung Cancer: Artificial Intelligence, Synergetics, Complex System Analysis, S...
 
Charaka Samhita Sutra Sthana 9 Chapter khuddakachatuspadadhyaya
Charaka Samhita Sutra Sthana 9 Chapter khuddakachatuspadadhyayaCharaka Samhita Sutra Sthana 9 Chapter khuddakachatuspadadhyaya
Charaka Samhita Sutra Sthana 9 Chapter khuddakachatuspadadhyaya
 
ANATOMY AND PHYSIOLOGY OF URINARY SYSTEM.pptx
ANATOMY AND PHYSIOLOGY OF URINARY SYSTEM.pptxANATOMY AND PHYSIOLOGY OF URINARY SYSTEM.pptx
ANATOMY AND PHYSIOLOGY OF URINARY SYSTEM.pptx
 
BRACHYTHERAPY OVERVIEW AND APPLICATORS
BRACHYTHERAPY OVERVIEW  AND  APPLICATORSBRACHYTHERAPY OVERVIEW  AND  APPLICATORS
BRACHYTHERAPY OVERVIEW AND APPLICATORS
 
Surat @ℂall @Girls ꧁❤8527049040❤꧂@ℂall @Girls Service Vip Top Model Safe
Surat @ℂall @Girls ꧁❤8527049040❤꧂@ℂall @Girls Service Vip Top Model SafeSurat @ℂall @Girls ꧁❤8527049040❤꧂@ℂall @Girls Service Vip Top Model Safe
Surat @ℂall @Girls ꧁❤8527049040❤꧂@ℂall @Girls Service Vip Top Model Safe
 
HOT NEW PRODUCT! BIG SALES FAST SHIPPING NOW FROM CHINA!! EU KU DB BK substit...
HOT NEW PRODUCT! BIG SALES FAST SHIPPING NOW FROM CHINA!! EU KU DB BK substit...HOT NEW PRODUCT! BIG SALES FAST SHIPPING NOW FROM CHINA!! EU KU DB BK substit...
HOT NEW PRODUCT! BIG SALES FAST SHIPPING NOW FROM CHINA!! EU KU DB BK substit...
 
Superficial & Deep Fascia of the NECK.pptx
Superficial & Deep Fascia of the NECK.pptxSuperficial & Deep Fascia of the NECK.pptx
Superficial & Deep Fascia of the NECK.pptx
 
Prix Galien International 2024 Forum Program
Prix Galien International 2024 Forum ProgramPrix Galien International 2024 Forum Program
Prix Galien International 2024 Forum Program
 
THOA 2.ppt Human Organ Transplantation Act
THOA 2.ppt Human Organ Transplantation ActTHOA 2.ppt Human Organ Transplantation Act
THOA 2.ppt Human Organ Transplantation Act
 
Physiology of Special Chemical Sensation of Taste
Physiology of Special Chemical Sensation of TastePhysiology of Special Chemical Sensation of Taste
Physiology of Special Chemical Sensation of Taste
 
MANAGEMENT OF ATRIOVENTRICULAR CONDUCTION BLOCK.pdf
MANAGEMENT OF ATRIOVENTRICULAR CONDUCTION BLOCK.pdfMANAGEMENT OF ATRIOVENTRICULAR CONDUCTION BLOCK.pdf
MANAGEMENT OF ATRIOVENTRICULAR CONDUCTION BLOCK.pdf
 
24 Upakrama.pptx class ppt useful in all
24 Upakrama.pptx class ppt useful in all24 Upakrama.pptx class ppt useful in all
24 Upakrama.pptx class ppt useful in all
 
Charaka Samhita Sutra sthana Chapter 15 Upakalpaniyaadhyaya
Charaka Samhita Sutra sthana Chapter 15 UpakalpaniyaadhyayaCharaka Samhita Sutra sthana Chapter 15 Upakalpaniyaadhyaya
Charaka Samhita Sutra sthana Chapter 15 Upakalpaniyaadhyaya
 
Evaluation of antidepressant activity of clitoris ternatea in animals
Evaluation of antidepressant activity of clitoris ternatea in animalsEvaluation of antidepressant activity of clitoris ternatea in animals
Evaluation of antidepressant activity of clitoris ternatea in animals
 
heat stroke and heat exhaustion in children
heat stroke and heat exhaustion in childrenheat stroke and heat exhaustion in children
heat stroke and heat exhaustion in children
 
Maxilla, Mandible & Hyoid Bone & Clinical Correlations by Dr. RIG.pptx
Maxilla, Mandible & Hyoid Bone & Clinical Correlations by Dr. RIG.pptxMaxilla, Mandible & Hyoid Bone & Clinical Correlations by Dr. RIG.pptx
Maxilla, Mandible & Hyoid Bone & Clinical Correlations by Dr. RIG.pptx
 
Surgical Site Infections, pathophysiology, and prevention.pptx
Surgical Site Infections, pathophysiology, and prevention.pptxSurgical Site Infections, pathophysiology, and prevention.pptx
Surgical Site Infections, pathophysiology, and prevention.pptx
 
Hemodialysis: Chapter 3, Dialysis Water Unit - Dr.Gawad
Hemodialysis: Chapter 3, Dialysis Water Unit - Dr.GawadHemodialysis: Chapter 3, Dialysis Water Unit - Dr.Gawad
Hemodialysis: Chapter 3, Dialysis Water Unit - Dr.Gawad
 

PPT Advance Trauma Life Support edisi 10

  • 1.
  • 2.
  • 3. Course Overview 3 of 21 Welcome and Introductions • Course Director • Faculty • Course Coordinator
  • 4. Course Overview 4 of 21 Welcome and Introductions We would like you to introduce yourselves now: Who are you? What is your professional background? What is your experience with trauma? What do you hope to gain from this course?
  • 5. Course Overview 5 of 21 Safe Learning Environment • Interactive, unfolding case discussions, stimulus questions • Participation required • Respect for each other • Try answering the questions – it’s ok if you get it wrong
  • 6. Course Overview 6 of 21 M 43 year old male, driver MVC, involved in head on collision with truck I Right sided bruising and abrasions on Chest, deformed right leg, S Non responsive, RR 30, HR 130, BP 80/60 T On spine board with C spine collar Discussion Question: What are your concerns with this patient?
  • 7. Course Overview 7 of 21 The ATLS course provides one acceptable method for the safe, immediate management of trauma patients.
  • 8. Course Overview 8 of 21 Program Goals ATLS course provides participants with a safe and reliable method to: 1. Assess a patient’s condition rapidly and accurately. 2. Resuscitate and stabilize patients according to priority. 3. Determine whether a patient’s needs exceed a facility’s resources and/or provider’s capabilities. 4. Arrange transfer when indicated A ?⃝ → Oxygenation flow to cells ⑦ → eoagulophaty
  • 9. Course Overview 9 of 21 Course Objectives Upon completion of the ATLS student course, you will be able to: 1. Demonstrate the concepts and principles of the primary and secondary patient assessments. 2. Establish management priorities in the initial management of a trauma patient. 3. Initiate the primary and secondary management of a simulated trauma patient in a timely manner. 4. In a given trauma simulation, demonstrate the skills that are often required in the initial assessment and treatment of patients with multiple injuries. Airway → 5 ppnypbab → baca buta TABCDE
  • 10. Course Overview 10 of 21 The Need • 5.8 million people die every year from unintentional injury and violence -- more than nine people every minute. • Injury accounts for 18% of the world’s burden of disease. • Motor vehicle crashes alone cause more than 1 million deaths annually and 20 to 50 million significant injuries. Self-inflicted violence 16% Interpersonal violence 10% Other 17% Road traffic injuries 25% Fires 5% Poisoning 6% Falls 6% War 6% Drowning 9%
  • 11. Course Overview 11 of 21 ATLS provides a common language
  • 12. Course Overview 12 of 21 The Beginning
  • 13. Course Overview 13 of 21 “When I can provide better care in the field with limited resources than what my children and I received at the primary care facility, there is something wrong with the system, and the system has to be changed.” James Styner, MD, FACS 1977
  • 14. Course Overview 14 of 21 Trimodal to Bimodal Distribution L % Jam → 5 days death 1 JM → besok 71 Jan → hari in
  • 15. Course Overview 15 of 21 ATLS Concept • Follow ABCDE approach to evaluation and treatment. • Treat the greatest threat to life first. • Recognize the definitive diagnosis is not immediately important. • Understand that time is of the essence. • Do no further harm. ✗ ray thorax e pelvis
  • 16. Course Overview 16 of 21 ATLS Concept Airway with restriction of cervical spine motion Breathing and ventilation Circulation with hemorrhage control Disability: Neurological status Exposure / Environmental control Bleeding 4) I II IT IV ≤750 ≤ 1500 ≤2000712000
  • 17. Course Overview 17 of 21 Initial Assessment and Management
  • 18. Course Overview 18 of 21 ATLS Educational Format • Introductory lecture • Interactive group discussions • Interactive skill sessions • Simulated patient scenarios • Written examinations • mATLS online learning modules • MyATLS mobile app
  • 19. Course Overview 19 of 21 International ATLS Program • 83 countries • 3,380 courses • 68,000 students • MyATLS mobile app 181 countries, 216,000 downloads
  • 20. Course Overview 20 of 21 Impact of ATLS Program • Documented improvement in the care of injured patients after implementation of program • Organized trauma care resulting in reduced injury mortality • Retention of organizational and procedural skills
  • 21. Course Overview 21 of 21 Any Questions? ABC → fangs, wana Jaknnya Coagulopathy → ttoas of AS/dos's Death hepotrensi ↳ Korans Imprint → 6 man't ↓ Man blasts Kamata Kling
  • 22. Course Overview 22 of 21 Summary • ABCDE approach to trauma care • Do no further harm • Treat the greatest threat to life first • One safe way • A common language
  • 23. Course Overview 23 of 21 Video 1
  • 25. Course Overview 25 of 21 1 Initial Assessment and Management The primary survey (ABCD) is the cornerstone of the initial assessment of the trauma patient. Repeat the primary survey frequently to identify any deterioration in the patient’s status that indicates the need for additional intervention.
  • 26. Course Overview 26 of 21 Objectives By the end of this interactive discussion, you will be able to: 1. Explain the importance of preparation prior to trauma patient arrival. 2. Evaluate the mechanism of injury to determine the patient’s potential injuries. 3. Identify the correct sequence of priorities for the assessment of a multiply injured patient. 4. Apply the principles of the primary and secondary surveys to the assessment of a multiply injured patient. 5. Discuss the importance of reevaluating a patient who is not responding appropriately to initial resuscitation and management. 6. Recognize patients who require transfer to another facility for definitive management.
  • 27. Course Overview 27 of 21 1 Case Scenario 18-year-old male , unrestrained driver in MVC vs. tree None reported Vitals not reported Prolonged extrication; transported to ED by ambulance; O2 by mask; fluids via single IV; spinal motion restricted on long spine board Initial Assessment and Management M I S T
  • 28. Course Overview 28 of 21 1 Discussion Questions: 1. How would you prepare for the arrival of this patient? 2. What other information would be helpful to know in order to prepare? 3. From the history, what are the potential injuries this patient may have suffered? Initial Assessment and Management Case Details M 18-year-old male , unrestrained driver in MVC vs. tree None reported Vitals not reported Prolonged extrication; transported to ED by ambulance; O2 by mask; fluids via single IV; spinal motion restricted on long spine board I S T
  • 29. Course Overview 29 of 21 1 Case Scenario Progression • EMS report: patient is lethargic, mumbling unintelligibly • Patient has facial injuries • Vital signs: HR 120; BP 90/40; RR 24, O2 sat 89%, temp 36°C Initial Assessment and Management
  • 30. Course Overview 30 of 21 1 Discussion Questions: 1. Based on this information, what interventions can be done in the prehospital setting? 2. Which patients should be immediately transported to the trauma center based on their field presentation? Initial Assessment and Management Case Details • EMS report: patient is lethargic, mumbling unintelligibly • Patient has facial injuries • Vital signs: HR 120; BP 90/40; RR 24, O2 sat 89%, temp 36°C
  • 31. Course Overview 31 of 21 1 Case Scenario Progression • Patient arrives at hospital • Vital signs: HR 120; BP 90/palp; RR 20; O2 sat 82%, temp 35.5°C. Initial Assessment and Management
  • 32. Course Overview 32 of 21 1 Discussion Questions: 1. What are your clinical concerns? 2. What are your management priorities? Initial Assessment and Management Case Details • Patient arrives at hospital • Vital signs: HR 120; BP 90/palp; RR 20; O2 sat 82%, temp 35.5°C.
  • 33. Course Overview 33 of 21 1 Case Scenario Progression Primary survey reveals: A: Obvious facial trauma and mumbling incoherently. B: Decreased breath sounds, L chest; no visible neck veins C: Minimal bleeding; open L femur fracture; L chest bruising; possible pelvic fracture D: Localizes to pain with upper extremities; moans to painful stimuli; does not open eyes Initial Assessment and Management tmpt mpnanpurg - thorax ¢ Caution abdomm _ retroperitoneal darah (Prius hulas Panjang
  • 34. Course Overview 34 of 21 1 Initial Assessment and Management Case Details A: Obvious facial trauma and mumbling incoherently. B: Decreased breath sounds, L chest; no visible neck veins C: Minimal bleeding; open L femur fracture; L chest bruising; possible pelvic fracture D: Localizes to pain with upper extremities; moans to painful stimuli; does not open eyes Discussion Questions: 1. What are your clinical concerns? 2. What are your management priorities? -
  • 35. Course Overview 35 of 21 1 Case Scenario Progression • Patient intubated • Femur fracture reduced and immobilized; pelvic stabilizing device applied • 500 mL warmed crystalloid and 1 unit unmatched pRBCs IV • Vital signs: HR 97; BP 110/64; RR 24; O2 sat 96% • Patient begins to respond to verbal stimuli, opens eyes, and tries to brush away your hands Initial Assessment and Management ppnunsong yg primary dow - chest ✗ ray ( ppluic ✗- ray
  • 36. Course Overview 36 of 21 1 Discussion Questions: 1. What additional adjuncts and treatments would you order at this time? 2. When should the transfer occur and what tests are necessary before transferring the patient? Initial Assessment and Management Case Details • Patient intubated • Femur fracture reduced and immobilized; pelvic stabilizing device applied • 500 mL warmed crystalloid and 1 unit unmatched pRBCs IV • Vital signs: HR 97; BP 110/64; RR 24; O2 sat 96% • Patient begins to respond to verbal stimuli, opens eyes, and tries to brush away your hands
  • 37. Course Overview 37 of 21 1 Case Scenario Progression • Patient’s LOC decreases • Patient opens his eyes to pressure and moves away from stimulus (normal flexion) • Vital signs: HR 100; BP 100/60; RR 20 • Good breath sounds bilaterally Initial Assessment and Management
  • 38. Course Overview 38 of 21 Secondary survey: • Pupils: 5 mm, minimally reactive, L; 6 mm, reactive, R • Laceration and soft tissue injury, L temporal-frontal region; no active bleeding • L hemotympanum • Large ecchymosis, L anterior chest • Abdomen soft, nondistended 1 Initial Assessment and Management f- Syarat → ☒ stabil → ABC stab - → fast⊕ pprw
  • 39. Course Overview 39 of 21 1 Discussion Question: What is your first step when a patient’s condition changes? Initial Assessment and Management ↳ Reevaluation
  • 40. Course Overview 40 of 21 1 Discussion Question: When does the secondary survey occur, and how is it conducted? Initial Assessment and Management
  • 41. Course Overview 41 of 21 1 Case Scenario Progression • No neurosurgery on site • Decision: transfer patient to another facility for definitive care • Contact the family to give update and obtain consent for transfer Initial Assessment and Management
  • 42. Course Overview 42 of 21 1 Discussion Questions: 1. The family insists on obtaining a CT of the head, even though this will significantly delay transport (the team is ready). Do you agree and why? 2. What information should you provide to the receiving facility? Initial Assessment and Management Case Details • Patient intubated • Femur fracture reduced and immobilized; pelvic stabilizing device applied • 500 mL warmed crystalloid and 1 unit unmatched pRBCs IV • Vital signs: HR 97; BP 110/64; RR 24; O2 sat 96% • Patient begins to respond to verbal stimuli, opens eyes, and tries to brush away your hands fdrnhtas → Mist - Mtkhonism spring treatment →tovaoasl tvrdakcn
  • 43. Course Overview 43 of 21 1 Case Scenario Conclusion The patient is transferred to a trauma center via air, and goes to surgery for evacuation of an intracranial hematoma. Initial Assessment and Management
  • 44. Course Overview 44 of 21 1 Any Questions? Initial Assessment and Management
  • 45. Course Overview 45 of 21 1 Review Objectives By the end of this interactive discussion, you will be able to: 1. Explain the importance of preparation prior to trauma patient arrival. 2. Evaluate the mechanism of injury to determine the patient’s potential injuries. 3. Identify the correct sequence of priorities for the assessment of a multiply injured patient. 4. Apply the principles of the primary and secondary surveys to the assessment of a multiply injured patient. 5. Discuss the importance of reevaluating a patient who is not responding appropriately to initial resuscitation and management. 6. Recognize patients who require transfer to another facility for definitive management. Initial Assessment and Management
  • 46. Course Overview 46 of 21 Key Learning Points 1. The initial management of the injured patient requires: • coordination with prehospital providers • preparation for receiving the patient • anticipation of injuries based on the mechanism of injury 2. The evaluation of all trauma patients follows a precise algorithm. 3. Patients who exceed the capability of the institution should be identified rapidly and process for transfer begun. 4. Evaluate the patient according to priority using the ABCDEs. 1 Initial Assessment and Management
  • 47. Course Overview 47 of 21 1 Initial Assessment and Management Video 2
  • 48. Course Overview 48 of 21 1 Initial Assessment and Management Video 3
  • 50. 2 Airway and Ventilatory Management The earliest priorities in managing the injured patient are to ensure an intact airway and recognize a compromised airway. pg [ Korma balk → obspruasi ] Rppvaluasi ↓ [ Suara Sevak {⊕¥%Éah momaxai dat bantu nafas protrusion " "' Sam poi to -both ④ ada Cpdera
  • 51. Objectives By the end of this interactive discussion, you will be able to: 1. Identify different clinical situations in which airway compromise is likely to occur. 2. Recognize the signs and symptoms of acute airway compromise in a trauma case scenario. 3. Determine factors that may lead to a difficult airway. 4. Apply the ATLS airway algorithm to a case scenario involving a patient with a difficult airway. 5. Define the term definitive airway. 2 Airway and Ventilatory Management → tubp dalam trachea dgn baton dlkoimbongkan
  • 52. Case Scenario 43-year-old obese- restrained driver lost control of his small vehicle while traveling at a high speed on an icy road; crashed driver’s side into a large tree. None reported Patient combative during extrication Spinal motion restricted on long spine board; C collar; bag-mask ventilation M I S T 2 Airway and Ventilatory Management
  • 53. Discussion Questions: 1. What aspects of the reported mechanism of injury present a risk of airway compromise? 2. Which clinical findings suggest(s) potential airway compromise? Case Details M 43-year-old obese- restrained driver lost control of his small vehicle while traveling at high speed on an icy road; crashed driver’s side into a large tree. None reported Patient combative during extrication Spinal motion restricted on long spine board; C collar; bag- mask ventilation I S T 2 Airway and Ventilatory Management Multitrauma → maxi/ofaaal , caracal trauma , alcohol combative Cmplawon )
  • 54. Discussion Questions: 3. How do you know if the patient’s airway is patent? 4. What are some patient factors that may contribute to a difficult airway in this patient? Case Details M 43-year-old obese- restrained driver lost control of his small vehicle while traveling at high speed on an icy road; crashed driver’s side into a large tree. None reported Patient combative during extrication Spinal motion restrictedon long spine board; C collar; bag- mask ventilation I S T 2 Airway and Ventilatory Management Komunikass dgn balk → ✗ blsa → 19 mungklna airway④ Stahl Obpshtas & trauma
  • 55. Discussion Questions: 5. Are there additional factors that may be present in other trauma patients? 6. How might we predict a difficult airway? Case Details M 43-year-old obese- restrained driver lost control of his small vehicle while traveling at high speed on an icy road; crashed driver’s side into a large tree. None reported Patient combative during extrication Spinal motion restricted on long spine board; C collar; bag- mask ventilation I S T 2 Airway and Ventilatory Management Extreme agp , ☆ Pdcatric i Brosnanf ↳mon ~
  • 56. Case Scenario Progression On arrival: • Patient lethargic • Asymmetric chest expansion • Shallow breathing, O2 sat 82% • Significant facial injuries, L chest bruising You are attempting to assist his ventilation with bag-mask ventilation. 2 Airway and Ventilatory Management
  • 57. Discussion Questions: 1. What are the symptoms of inadequate ventilation? 2. What are the signs of inadequate ventilation? 3. What are your next steps in management? 4. What equipment is required? • Patient lethargic • Asymmetric chest expansion • Shallow breathing, O2 sat 82% • Significant facial injuries, L chest bruising • You are attempting to assist his ventilation with bag-mask ventilation. Case Details 2 Airway and Ventilatory Management bpbaslcan salon natas → A da - haha → mug, f
  • 58. Case Scenario Progression • Patient now unconscious • Vital signs: systolic BP 100; HR 120 • Upper airway suctioned, some bloody sputum cleared • Teeth and facial bones intact • Oral airway inserted and bag-mask ventilation continued with 100% oxygen. O2 sat improves to 93% • You decide to intubate the patient 2 Airway and Ventilatory Management
  • 59. Discussion Questions: 1. What constitutes a definitive airway? 2. How do you prepare for this intubation? 3. What adjuncts might be used during intubation? Case Details • Patient now unconscious • Vital signs: systolic BP 100; HR 120 • Upper airway suctioned, some bloody sputum cleared • Teeth and facial bones intact • Oral airway inserted and bag-mask ventilation continued with 100% oxygen. O2 sat improves to 93% • You decide to intubate the patient 2 Airway and Ventilatory Management
  • 60. Case Scenario Progression • You have difficulty intubating the patient. • LMA is inserted, but it is difficult to get an adequate seal. • An attempt with a gum elastic bougie is successful. • The patient oxygenation improves. 2 Airway and Ventilatory Management
  • 61. Discussion Questions: 1. When is a surgical airway indicated? Should one have been performed now 2. Is there anything that may have been considered for intubation prior to the patient’s decompensation? Case Details • You have difficulty intubating the patient. • LMA is inserted, but it is difficult to get an adequate seal. • Successful drug-assisted intubation performed using a gum elastic bougie. • The patient oxygenation improves. 2 Airway and Ventilatory Management extreme trauma meal Cofaacal
  • 62. Case Scenario Progression • Following successful drug-assisted intubation: • Vital signs a few minutes following intubation: HR 130; BP 90/30; O2 sat 70%. • Breath sounds are diminished on the L side. 2 Airway and Ventilatory Management - Evalbass Ett → false - trauma - homothorax l pneumothorax
  • 63. Discussion Questions: 1. What are the possible causes of this patient’s deterioration? 2. How can you diagnose and treat these possible causes? Case Details • Successful drug-assisted intubation performed • Vital signs a few minutes following intubation: HR 130; BP 90/30; O2 sat 70%. • Breath sounds are diminished on the L side. 2 Airway and Ventilatory Management Somnath ? M { look 5 Mkt hiding Phngnmbangon dada gurgling Menial airway { ""m → Suara haters abnormal ✗snoomg stridor HM % hcembusan ragas
  • 64. Case Scenario Conclusion • Right mainstem intubation is discovered and readjusted. • Patient’s vital signs return to normal. • Primary survey is completed. 2 Airway and Ventilatory Management
  • 65. Any Questions? 2 Airway and Ventilatory Management
  • 66. Objectives By the end of this interactive discussion, you will be able to: 1. Identify different clinical situations in which airway compromise is likely to occur. 2. Recognize the signs and symptoms of acute airway compromise in a trauma case scenario. 3. Determine factors that may lead to a difficult airway. 4. Apply the ATLS airway algorithm to a case scenario involving a patient with a difficult airway. 5. Define the term definitive airway. 2 Airway and Ventilatory Management Timon
  • 67. Key Learning Points 1. One of earliest priorities is recognizing a compromised airway. 2. All trauma patients should receive supplemental oxygen. 3. Risk of airway compromise and difficult airway management can be predicted. 4. Alterations in mental status (agitation, combativeness, confusion, or obtundation) may indicate the need for airway management. 5. A definitive airway (cuffed tube in trachea below vocal cords) should be obtained in cases of airway compromise. 2 Airway and Ventilatory Management
  • 68. Skill Station : Basic and Advanced Airway Management
  • 78.
  • 79. 79 of 17 Shock 3 The first step in the initial management of shock is to recognize its presence. The diagnosis of shock is based on clinical recognition of the presence of inadequate tissue perfusion and oxygenation.
  • 80. 80 of 17 Shock 3 Objectives By the end of this interactive discussion, you will be able to: 1. Apply the ATLS principles to the management of a trauma patient with shock. 2. Recognize the signs and symptoms of shock. 3. Evaluate a patient case scenario to determine the possible causes of shock. 4. Discuss the changes that may be seen on initial investigations of a patient with shock. 5. Evaluate the efficacy of initial fluid management of a patient in shock. 6. Discuss the impact of special patient factors on the management of shock.
  • 81. 81 of 17 Shock 3 Case Scenario 80-year-old male, unrestrained passenger in a low-speed MVC None reported Patient confused Brought to ED by paramedics M I S T
  • 82. 82 of 17 Shock 3 Discussion Questions: 1. What are the possible reasons for the patient’s confusion? 2. What additional scene information would be helpful to obtain from the prehospital providers that could help you differentiate the causes of his confusion? Case Details M 80-year-old male, unrestrained passenger in a low- speed MVC None reported Patient confused Brought to ED by paramedics I S T
  • 83. 83 of 17 Shock 3 Case Scenario Progression • Vital signs: BP 100/70; HR 100; RR 20 • Patient on a stretcher, receiving IV fluids
  • 84. 84 of 17 Shock 3 Discussion Questions: 1. Based on the information given, is this patient in shock? What additional information is needed to help determine this? 2. What vital signs and laboratory studies support the diagnosis of shock? 3. Can a single vital sign or laboratory result diagnose shock? • Vital signs: BP 100/70; HR 100; RR 20 • Patient on a stretcher, receiving IV fluids Case Details
  • 85. 85 of 17 Shock 3 Discussion Questions: 4. What is the most common cause of shock in a trauma patient? 5. What types of soft tissue or bony injuries might result in shock? • Vital signs: BP 100/70; HR 100; RR 20 • Patient on a stretcher, receiving IV fluids Case Details
  • 86. 86 of 17 Shock 3 Case Scenario Progression • Two large-bore IVs placed, 1 L crystalloid bolus given • Vital signs post treatment: BP 98/77; HR 80 • The patient remains confused and unable to give his medical history
  • 87. 87 of 17 Shock 3 Discussion Questions: 1. What adjuncts should be considered to determine the cause of shock (if not already done)? 2. How should further resuscitation proceed? 3. How will you continue to monitor this patient’s ongoing response to fluid resuscitation? Case Details • Two large-bore IVs placed, 1 L crystalloid bolus given • Vital signs post treatment: BP 98/77; HR 80 • The patient remains confused and unable to give his medical history
  • 88. 88 of 17 Shock 3 Case Scenario Progression • Chest x-ray shows multiple rib fractures, large R hemothorax • The patient’s family arrives and provides history: o coronary artery disease o coronary artery stenting 1 year ago o Meds: beta blocker, clopidogrel (Plavix), and aspirin
  • 89. 89 of 17 Shock 3 Discussion Questions: 1. How do the beta blockers affect this patient’s presentation and response to interventions? 2. What management concerns are presented by the antiplatelet agents the patient is taking? 3. What medical condition could further impact the patient’s response to shock? Case Details • Chest x-ray shows multiple rib fractures, large R hemothorax • Patient history: o coronary artery disease o coronary artery stenting 1 year ago o Meds: beta blocker, clopidogrel (Plavix), and aspirin
  • 90. 90 of 17 Shock 3 Case Scenario Conclusion • Platelet transfusion initiated. • R chest tube is placed 750 mL of blood obtained • Subsequent chest tube output is 50 mL/2 hours
  • 91. 91 of 17 Shock 3 Any Questions?
  • 92. 92 of 17 Shock 3 Review Objectives By the end of this interactive discussion, you will be able to: 1. Apply the ATLS principles to the management of a trauma patient with shock. 2. Recognize the signs and symptoms of a trauma patient in shock. 3. Evaluate a patient case scenario to determine the possible causes of shock. 4. Discuss the changes that may be seen on initial investigations of a patient with shock. 5. Evaluate the efficacy of initial fluid management of a patient in shock. 6. Discuss the impact of special patient factors on the management of shock.
  • 93. 93 of 17 Shock 3 Key Learning Points 1. Hemorrhage is the most common cause of shock after injury. 2. No single laboratory test and no single vital sign on its own can diagnose shock. 3. Massive blood loss may produce only minimal acute decrease in hemoglobin or hematocrit. 4. Major soft tissue injuries and fractures can be associated with significant hemorrhage. 5. The patient’s response to initial fluid therapy will help guide subsequent therapy. 6. A variety of special conditions may affect the patient’s response to shock and the management of it (e.g., age, medication use).
  • 95. 95 of 17 Shock 3 Thoracic injury is common in polytrauma patients and can be life-threatening, especially if not promptly identified and treated during the primary survey. 4 Thoracic Trauma
  • 96. 96 of 17 Shock 3 Objectives By the end of this interactive discussion, you will be able to: 1. Apply the ATLS principles to the management of a patient with thoracic trauma. 2. Recognize the important life-threatening injuries in a patient with thoracic trauma. 3. Evaluate the case scenario of a patient with thoracic trauma to identify immediate life- threatening injuries. 4. Discuss the clinical findings and adjunctive studies that may be useful during the secondary survey in a patient with thoracic trauma. 4 Thoracic Trauma
  • 97. 97 of 17 Shock 3 Case Scenario 27-year-old male unrestrained driver in high-speed, frontal-impact MVC Airway patent, obvious respiratory distress BP 90/50; HR 110; RR 36; and GCS 14 None reported M I S T 4 Thoracic Trauma
  • 98. 98 of 17 Shock 3 Discussion Questions: 1. What life-threatening injuries might one discover in the primary survey that could account for the patient’s clinical status? 2. What are the next steps in the evaluation and treatment of this patient? Case Details M 27-year-old male unrestrained driver in high-speed, frontal-impact MVC Airway patent, obvious respiratory distress BP 90/50; HR 110; RR 36; and GCS 14 None reported I S T 4 Thoracic Trauma
  • 99. 99 of 17 Shock 3 Discussion Questions: 1. What are the immediately life-threatening thoracic injuries involving: • Airway • Breathing • Circulation Case Details M 27-year-old male unrestrained driver in high-speed, frontal-impact MVC Airway patent, obvious respiratory distress BP 90/50; HR 110; RR 36; and GCS 14 None reported I S T 4 Thoracic Trauma
  • 100. 100 of 17 Shock 3 Case Scenario Progression On arrival to ED: • Patient complains of profound shortness of breath, asking to sit up • O2 sat 89%. • Cervical collar in place • Trachea deviated to left • Breath sounds absent on left • Heart sounds normal • Left chest wall crepitus 4 Thoracic Trauma
  • 101. 101 of 17 Shock 3 Discussion Questions: 1. What is this patient’s most likely diagnosis? 2. What is the appropriate technique to alleviate this patient’s condition? • Patient complains of profound shortness of breath, asking to sit up • O2 sat 89%. • Cervical collar in place • Trachea deviated to left • Breath sounds absent on left • Heart sounds normal • Left chest wall crepitus Case Details 4 Thoracic Trauma
  • 102. 102 of 17 Shock 3 Case Scenario Progression • Needle decompression performed, no rush of air • Vital signs unchanged 4 Thoracic Trauma
  • 103. 103 of 17 Shock 3 Discussion Question: What alternative management strategy can you use? Case Details • Needle decompression performed, no rush of air • Vital signs unchanged 4 Thoracic Trauma
  • 104. 104 of 17 Shock 3 Discussion Question: What are the differences in clinical presentation between a tension pneumothorax and an open pneumothorax? 4 Thoracic Trauma
  • 105. 105 of 17 Shock 3 Case Scenario Progression • Finger thoracostomy performed • Vital signs: RR 28; HR 110; BP 100/60 • Thoracostomy tube placed, 600 mL dark blood drained • Two large-bore IV lines established, isotonic fluid given • Type and crossmatch requested • Chest x-ray shows: o obscured L diaphragm o multiple L rib fractures o wide mediastinum o pulmonary contusion 4 Thoracic Trauma
  • 106. 106 of 17 Shock 3 Discussion Questions: 1. What are the indications for operation in a patient with traumatic hemothorax? 2. What resuscitative measures should be undertaken in a patient with massive hemothorax? 4 Thoracic Trauma
  • 107. 107 of 17 Shock 3 Discussion Questions: 3. What other potential life threats might exist in a patient with this mechanism? For each, what would the clinical presentation/findings and the appropriate treatment be? 4. What test would you perform to make the diagnosis during the secondary survey? • Finger thoracostomy performed • Vital signs: RR 28; HR 110; BP 100/60 • Thoracostomy tube placed, 600 mL dark blood drained • Two large-bore IV lines established, isotonic fluid given • Type and crossmatch requested • Chest x-ray shows: • obscured L diaphragm • multiple L rib fractures • wide mediastinum • pulmonary contusion Case Details 4 Thoracic Trauma
  • 108. 108 of 17 Shock 3 Potential Life Threat Clinical Presentation/Findings Treatment Pitfalls Simple Pneumothorax +/- shortness of breath No hypotension Diagnosis by chest x-ray Chest tube drainage Could become tension pneumothorax if untreated Hemothorax Dullness to percussion Diagnosis by chest x-ray Chest tube drainage Could become massive hemothorax Flail Chest and Pulmonary Contusion May see paradoxical movement of chest wall More commonly presents with pain and poor respiratory excursions Oxygen Analgesia Intubation if necessary Progressive respiratory failure Blunt Cardiac Injury ECG changes Cardiac monitoring Therapy based on clinical status At risk for clinically significant dysrhythmias Traumatic Aortic Disruption May be asymptomatic Multiple possible radiographic findings Endovascular or open surgical repair Blood pressure control important prior to definitive therapy Traumatic Diaphragm Injury Respiratory distress Obscured left diaphragm border Evidence of abdominal viscera in chest Operative repair Concomitant pulmonary contusion may mask diaphragm injury Esophageal injury Chest pain; mediastinal air on imaging; crepitus delayed fever Operative repair Delayed diagnosis 4 Thoracic Trauma
  • 109. 109 of 17 Shock 3 Case Scenario Progression • 250 mL of IV fluids given • Vital signs: BP 110/70; HR 110; RR 18 • O2 sat 91% on nonrebreather mask 4 Thoracic Trauma
  • 110. 110 of 17 Shock 3 Discussion Question: Which of the following treatments is best for pulmonary contusion/flail chest? A. Beta blockers B. Massive fluid resuscitation C. Immediate nebulizer treatment D. Supplemental oxygen, pain control, and recognition of the potential for respiratory failure 4 Thoracic Trauma
  • 111. 111 of 17 Shock 3 Case Scenario Conclusion Your institution has the capability to care for this patient, and you order a CT scan for further evaluation. 4 Thoracic Trauma
  • 112. 112 of 17 Shock 3 Case Scenario #2 25-year-old male, high-speed MVC Awake and responds to questions, complaining of chest pain and shortness of breath, gurgling sounds L lung base. BP 102/76; HR 134; O2 sat 93% on oxygen by face mask Chest x-ray M I S T 4 Thoracic Trauma
  • 113. 113 of 17 Shock 3 Discussion Questions: 1. What abnormalities do you note on the chest film? 4 Thoracic Trauma
  • 114. 114 of 17 Shock 3 Discussion Questions: 2. What is the treatment for a traumatic diaphragm injury? 3. Aside from the tracheal deviation to the right seen on the chest film, what other signs and x-ray findings might one see in a patient with traumatic aortic disruption? Case Details M 25-year-old male, high-speed MVC Awake and responds to questions, complaining of chest pain and shortness of breath, gurgling sounds L lung base. BP 102/76; HR 134; O2 sat 93% on oxygen by face mask Chest x-ray I S T 4 Thoracic Trauma
  • 115. 115 of 17 Shock 3 Case Scenario Progression CT scan shows blunt aortic injury 4 Thoracic Trauma
  • 116. 116 of 17 Shock 3 Discussion Questions: 1. What is the expected hemodynamic impact of a contained aortic disruption from blunt trauma? 2. If a patient with a known contained aortic disruption from blunt trauma becomes hypotensive, what should you consider? 3. What therapeutic steps should a clinician consider when managing a traumatic aortic disruption? Case Details • CT scan shows blunt aortic injury 4 Thoracic Trauma
  • 117. 117 of 17 Shock 3 Case Scenario Conclusion • Discussion with anesthesiology re: management of the patient’s aortic injury • Patient undergoes repair of his diaphragmatic injury via laparotomy • The following day, he undergoes endovascular repair of his aortic injury • He does well after 2 weeks in the ICU and is discharged home 4 Thoracic Trauma
  • 118. 118 of 17 Shock 3 Any Questions? 4 Thoracic Trauma
  • 119. 119 of 17 Shock 3 Review Objectives By the end of this interactive discussion, you will be able to: 1. Apply the ATLS principles to the management of a patient with thoracic trauma. 2. Recognize the important life-threatening injuries in a patient with thoracic trauma. 3. Evaluate the case scenario of a patient with thoracic trauma to identify immediate life- threatening injuries. 4. Discuss the clinical findings and adjunctive studies that may be useful during the secondary survey in a patient with thoracic trauma. 4 Thoracic Trauma
  • 120. 120 of 17 Shock 3 Key Learning Points 1. It is important to recognize thoracic life-threatening problems in polytrauma patients. 2. Most immediate thoracic life-threatening problems can be recognized without special testing and may be treated with: • airway control • decompression and/or • fluid resuscitation 3. Potential life-threatening problems can become immediate life-threatening problems if untreated (e.g., a simple pneumothorax can become a tension pneumothorax). 4 Thoracic Trauma
  • 122. 122 of 17 Shock 3 Breathing Video 1 (paradoxical chest wall movement) Respiratory Distress
  • 123. 123 of 17 Shock 3 Breathing Video 2 Chest Tube
  • 124. 124 of 17 Shock 3 Breathing Video 3 Respiratory Distress (Child)
  • 125. 125 of 17 Shock 3 Chest Tube (Child) Breathing Video 4
  • 126. 126 of 17 Shock 3 Chest Tube (Child) Breathing Video 5
  • 127. 127 of 17 Shock 3 Breathing Video 6 Dressing on a Sucking Chest Wound
  • 128. 5 Abdominal and Pelvic Trauma Tenth Edition
  • 129. 129 of 17 Shock 3 When uncontrolled or unrecognized, blood loss from abdominal and pelvic injuries can result in preventable death. 5 Abdominal and Pelvic Trauma
  • 130. 130 of 17 Shock 3 Objectives By the end of this interactive discussion, you will be able to: 1. Identify the anatomic regions of the abdomen that are critical in assessing and managing trauma patients. 2. Discuss the risk for abdominal and pelvic injuries based on the mechanism of injury. 3. Identify patients who require surgical consultation and possible surgical and/or catheter-based intervention. 4. Determine appropriate diagnostic procedures to ascertain if a patient has ongoing hemorrhage and/or other injuries that can cause delayed morbidity and mortality. 5. Formulate an acute management plan for abdominal and pelvic injuries utilizing a case scenario. 6. Discuss the importance of early identification and emergent management of pelvic hemorrhage. 5 Abdominal and Pelvic Trauma
  • 131. 131 of 17 Shock 3 Case Scenario 28-year-old male, helmeted motorcyclist, high-speed collision, head-on into the side of a vehicle that pulled out in front of him Patient reports brief loss of consciousness, complains of pain in chest, abdomen, and pelvis BP 100/75; HR 115; RR 20, and GCS 15 Backboard and c-collar M I S T 5 Abdominal and Pelvic Trauma
  • 132. 132 of 17 Shock 3 Discussion Questions: 1. What are your priorities for management of this patient? 2. What is your interpretation of the vital signs? 3. What is your initial therapy for this patient? Case Details M 28-year-old male, helmeted motorcyclist, high-speed collision, head-on into the side of a vehicle that pulled out in front of him Patient reports brief loss of consciousness, complains of pain in chest, abdomen, and pelvis BP 100/75; HR 115; RR 20, and GCS 15 Backboard and c-collar I S T 5 Abdominal and Pelvic Trauma
  • 133. 133 of 17 Shock 3 Case Scenario Progression • EMS reports: o Patient found 10 feet (3 meters) from his motorcycle o Patient lying on R side, wearing a helmet o Had been travelling at 45 mph (70 kph) • Patient reports: o Hard R sided landing, brief LOC o No allergies, no previous medical history or current medications 5 Abdominal and Pelvic Trauma
  • 134. 134 of 17 Shock 3 Discussion Question: 1. Based on the reported mechanism of injury, what intra-abdominal and/or pelvic injury is the patient likely to have sustained? • Patient found 10 feet (3 meters) from his motorcycle • Patient lying on R side, wearing a helmet • Had been travelling at 45 mph (70 kph) • Hard R sided landing, brief LOC • No allergies, no previous medical history or current medications Case Details 5 Abdominal and Pelvic Trauma
  • 135. 135 of 17 Shock 3 Discussion Questions: 1. How would the risk of intra- abdominal injury change if the patient described striking the handlebar into the epigastrium? 2. How would the risk of intra- abdominal injury change if a penetrating injury was observed? • Patient found 10 feet (3 meters) from his motorcycle • Patient lying on R side, wearing a helmet • Had been travelling at 45 mph (70 kph) • Hard R sided landing, brief LOC • No allergies, no previous medical history or current medications Case Details 5 Abdominal and Pelvic Trauma
  • 136. 136 of 17 Shock 3 Case Scenario Progression On examination: • Right-sided lower chest tenderness • Contusions, R chest, abdomen, and flank • Tender R upper quadrant, R flank, and suprapubic region • Pain on palpation of the anterior pelvis • No blood at the urethral meatus • Rectal examination is normal. • Vital signs following 500 ml of crystalloid solution: BP 110/75; HR 100; RR 20; GCS 15 5 Abdominal and Pelvic Trauma
  • 137. 137 of 17 Shock 3 Discussion Questions: 1. How should you assess the abdomen and pelvis for injury and as potential sources of bleeding? 2. Based on your knowledge of anatomy, the mechanism of injury, and this patient’s physical examination, what abdominal and/or pelvic injuries are most likely? 3. Is a FAST exam indicated at this stage? • Right-sided lower chest tenderness • Contusions, R chest, abdomen, and flank • Tender R upper quadrant, R flank, and suprapubic region • Pain on palpation of the anterior pelvis • No blood at the urethral meatus • Rectal examination is normal. • Vital signs following 500 ml of crystalloid solution: BP 110/75; HR 100; RR 20; GCS 15 Case Details 5 Abdominal and Pelvic Trauma
  • 138. 138 of 17 Shock 3 Discussion Question: 4. If this patient were female, what other examination would be relevant? 5. What radiological investigations would be appropriate to arrange now? • Right-sided lower chest tenderness • Contusions, R chest, abdomen, and flank • Tender R upper quadrant, R flank, and suprapubic region • Pain on palpation of the anterior pelvis • No blood at the urethral meatus • Rectal examination is normal. • Vital signs following 500 ml of crystalloid solution: BP 110/75; HR 100; RR 20; GCS 15 Case Details 5 Abdominal and Pelvic Trauma
  • 139. 139 of 17 Shock 3 Case Scenario Progression • Your institution has full surgical and radiological capabilities • Abdominal CT: grade III liver injury, R rib fractures, bilateral pelvic rami fractures • Surgical consultation obtained • Vital signs: BP normal; HR 100 • A total of 1.0 L of crystalloid solution given 5 Abdominal and Pelvic Trauma
  • 140. 140 of 17 Shock 3 Discussion Question: 1. Is emergent laparotomy warranted in this patient? 2. What clinical changes in this patient would indicate the need for operation, other therapies, or additional investigations regarding the abdominal and pelvic injuries? 3. How would your management change if the CT scan identified contrast extravasation suggesting bleeding in the pelvis? • Your institution has full surgical and radiological capabilities • Abdominal CT: grade III liver injury, R rib fractures, bilateral pelvic rami fractures • Surgical consultation obtained • Vital signs: BP normal; HR 100 • A total of 1.0 L of crystalloid solution given Case Details 5 Abdominal and Pelvic Trauma
  • 141. 141 of 17 Shock 3 Case Scenario Conclusion • Emergent laparotomy not required; nonoperative management undertaken • Patient admitted to ICU for monitoring, pain control, and respiratory care • Hemodynamics normal over 24 hours, transferred to the ward • Physical therapy for the pelvic fractures • Patient discharged home on day 6 5 Abdominal and Pelvic Trauma
  • 142. 142 of 17 Shock 3 Any Questions? 5 Abdominal and Pelvic Trauma
  • 143. 143 of 17 Shock 3 Review Objectives By the end of this interactive discussion, you will be able to: 1. Identify the anatomic regions of the abdomen that are critical in assessing and managing trauma patients. 2. Discuss the risk for abdominal and pelvic injuries based on the mechanism of injury. 3. Identify patients who require surgical consultation and possible surgical and/or catheter-based intervention. 4. Determine appropriate diagnostic procedures to ascertain if a patient has ongoing hemorrhage and/or other injuries that can cause delayed morbidity and mortality. 5. Formulate an acute management plan for abdominal and pelvic injuries utilizing a case scenario. 6. Discuss the importance of early identification and emergent management of pelvic hemorrhage. 5 Abdominal and Pelvic Trauma
  • 144. 144 of 17 Shock 3 Key Learning Points 1. Mechanism of injury is critical when considering abdominal and/or pelvic injury. 2. Thorough examinations of the chest, abdomen, and pelvis (anterior, lateral, posterior, and perineum) are required to avoid missing significant injuries. 3. Appropriate diagnostic procedures should be employed. 4. Surgical intervention is assessed via clinical findings and the patient’s response to management. 5. Early identification and emergent management of pelvic hemorrhage can be lifesaving. 5 Abdominal and Pelvic Trauma
  • 146. 146 of 17 Shock 3 Circulation Video 1 Packing an Open Wound
  • 147. 147 of 17 Shock 3 Circulation Video 2 Tourniquet
  • 148. 148 of 17 Shock 3 Circulation Video 3 Splint Traction
  • 149. 149 of 17 Shock 3 Splint Traction before traction after traction
  • 150. 150 of 17 Shock 3 Circulation Video 4 Intraosseous
  • 151. 151 of 17 Shock 3 Circulation Video 5 Pelvic Binder
  • 152. 152 of 17 Shock 3 Pelvic Binder before binder after binder Foto : pelvic diasthesis
  • 153. 153 of 17 Shock 3 Circulation Video 6 Central Venous Catheter
  • 155. 156 of 17 Shock 3 Circulation Video 9 Pericardiocentesis
  • 157. 158 of 17 Shock 3 The primary goal of treatment for patients with suspected traumatic brain injury is to prevent secondary brain injury. 6 Head Trauma
  • 158. 159 of 17 Shock 3 Objectives By the end of this interactive discussion, you will be able to: 1. Recognize the GCS score that corresponds to a severe head injury and indicates a comatose patient. 2. Identify the different types of intracranial bleeding seen on CT that are associated with traumatic brain injury. 3. Discuss the role of supplemental oxygen and systolic blood pressure maintenance in limiting secondary brain injury. 4. Describe the management of intracranial hypertension associated with the mass effect of blood or brain swelling. 5. Discuss the indications for early, rapid transfer to a center equipped to manage a patient with brain injury. 6 Head Trauma
  • 159. 160 of 17 Shock 3 Case Scenario 23-year-old male, fell from bicycle, hitting head on curb; no helmet 10 cm laceration to the L temporal-parietal region Initially able to say his name. HR 115; BP 100/60; O2 sat 88%; GCS 12 (E3V3M6) Two hours after transport to local hospital, patient has sonorous respirations; HR 120; BP 100/70; GCS 6 (E2V1M3) IV cannulas in situ, O2 via nasal prongs, 200mLs crystalloid infused M I S T 6 Head Trauma
  • 160. 161 of 17 Shock 3 Discussion Question: 1. What are the initial priorities in the management of this patient? 2. What are the signs that the patient’s injury is progressing? Case Details M 23-year-old male, fell from bicycle, hitting head on curb; no helmet 10 cm laceration to the L temporal-parietal region Initially able to say his name. HR 115; BP 100/60; O2 sat 88%; GCS 12 (E3V3M6) Two hours after transport to local hospital, patient has sonorous respirations; HR 120; BP 100/70; GCS 6 (E2V1M3) IV cannulas in situ, O2 via nasal prongs, 200mLs crystalloid infused I S T 6 Head Trauma
  • 161. 162 of 17 Shock 3 Case Scenario Progression • Patient intubated • Given 1 L normal saline • Vital signs: HR 100; BP 100/70; O2 Sat 94% 6 Head Trauma
  • 162. 163 of 17 Shock 3 Discussion Question: 1. How do you monitor this patient’s neurological status? 2. What other injuries and physical exam findings may suggest cranial and intracranial injury? • Patient intubated • Given 1 L normal saline • Vital signs: HR 100;BP 100/70; O2 Sat 94% Case Details 6 Head Trauma
  • 163. 164 of 17 Shock 3 Case Scenario Progression • Head, c-spine and abdominal CTs performed. • Head CT: temporal bone fracture, epidural hematoma, 1 cm of midline shift • C-spine normal 6 Head Trauma
  • 164. 165 of 17 Shock 3 Discussion Questions: 1. What types of intracranial hemorrhage can be identified on CT scan? 6 Head Trauma
  • 165. 166 of 17 Shock 3 Discussion Question: 2. What CT scan findings are indicative of severe head injury that may require intervention? • Head, c-spine and abdominal CTs performed. • Head CT: temporal bone fracture, epidural hematoma, 1 cm of midline shift • C-spine normal Case Details 6 Head Trauma
  • 166. 167 of 17 Shock 3 Case Scenario Progression • Thoracoabdominal CT scan normal • Initial management includes: o elevating the head of bed o sedation with short-acting medications o frequent neurological examinations 6 Head Trauma
  • 167. 168 of 17 Shock 3 Discussion Question: 1. What are the initial management options for this patient with severe brain injury and how do these differ from mild and moderate brain injury? 2. What are the indications for transferring a patient with a head injury to a center with a higher level of care? • Thoracoabdominal CT scan normal • Initial management includes: • elevating the head of bed • sedation with short-acting medications • frequent neurological examinations Case Details 6 Head Trauma
  • 168. 169 of 17 Shock 3 Case Scenario Progression • Neuro exam shows progression to extensor posturing. • Repeat CT scan shows new subdural hematoma with associated mass effect and midline shift. • Herniation appears imminent without treatment. • Patient requires a higher level of care and rapid transfer to neurosurgeon. 6 Head Trauma
  • 169. 170 of 17 Shock 3 Discussion Question: What are the initial treatment options that may protect the brain from ongoing swelling? • Neuro exam shows progression to extensor posturing. • Repeat CT scan shows new subdural hematoma with associated mass effect and midline shift. • Herniation appears imminent without treatment. • Patient requires a higher level of care and rapid transfer to neurosurgeon. Case Details 6 Head Trauma
  • 170. 171 of 17 Shock 3 Case Scenario Conclusion • Neurosurgeon recommends 0.5 g/kg mannitol and adjusting PaCO2 to 30 to 35 mm Hg. • Patient is immediately transported for emergency craniotomy. • Patient underwent successful evacuation of his intracranial hematoma. • He was discharged to a rehabilitation center for ongoing therapy. 6 Head Trauma
  • 171. 172 of 17 Shock 3 Any Questions? 6 Head Trauma
  • 172. 173 of 17 Shock 3 Review Objectives By the end of this interactive discussion, you will be able to: 1. Recognize the GCS score that corresponds to a severe head injury and indicates a comatose patient. 2. Identify the different types of intracranial bleeding seen on CT that are associated with traumatic brain injury. 3. Discuss the role of supplemental oxygen and systolic blood pressure maintenance in limiting secondary brain injury. 4. Describe the management of intracranial hypertension associated with the mass effect of blood or brain swelling. 5. Discuss the indications for early, rapid transfer to a center equipped to manage a patient with brain injury. 6 Head Trauma
  • 173. 174 of 17 Shock 3 Key Learning Points • GCS score is an objective, reproducible measurement of brain injury severity. • GCS of 8 or less is considered severe and indicative of a comatose patient. • Consider a CT scan of the head for any trauma patient with suspected traumatic brain injury. • Initial management of intracranial hypertension includes: • elevation of the head of bed • sedation • selective administration of mannitol and hypertonic saline 6 Head Trauma
  • 174. 175 of 17 Shock 3 Key Learning Points • Minimize secondary brain injury by: • adequate oxygenation (supplemental oxygen) • ensuring brain perfusion: SBP > 100 mm Hg (age 50-69) or > 110 mm Hg (15 – 49 and older than 70) • If no neurosurgical capability, consider early, rapid transfer 6 Head Trauma
  • 176. 177 of 17 Shock 3 Because spine injury can occur with both blunt and penetrating trauma, and with or without neurological deficits, it must be considered in all patients with multiple injuries. These patients require restriction of spinal motion to protect the spine from further damage until spine injury has been ruled out. 7 Spine and Spinal Cord Trauma
  • 177. 178 of 17 Shock 3 Objectives By the end of this interactive discussion, you will be able to: 1. Apply the ABC principles of ATLS when assessing a patient for spine injury. 2. Identify a common mechanism and type of spinal injury. 3. Describe the typical signs and symptoms of a patient with a spinal cord injury. 4. Describe the technique and importance of documentation of a potential spinal injury. 5. Describe the appropriate initial treatment of patients with spinal injuries. 6. Determine the appropriate disposition of patients with spine trauma. 7 Spine and Spinal Cord Trauma
  • 178. 179 of 17 Shock 3 Case Scenario 28-year-old male fell 5 meters (16 feet) from scaffolding, wearing hard hat; bystander reports patient landed head-first, with neck hyperextended None reported Skin warm; breathing shallow; not moving arms or legs; Vital signs: BP 80/62; HR 58; RR 28; GCS 15 None Reported M I S T 7 Spine and Spinal Cord Trauma
  • 179. 180 of 17 Shock 3 Discussion Questions: 1. What injuries has the patient likely sustained? 2. What other mechanisms of injury are associated with spinal cord trauma? 3. What types of shock are potentially present in this patient? Case Details M 28-year-old male fell 5 meters (16 feet) from scaffolding, wearing hard hat; bystander reports patient landed head- first, with neck hyperextended None reported Skin warm; breathing shallow; not moving arms or legs; Vital signs: BP 80/62; HR 58; RR 28; GCS 15 None Reported I S T 7 Spine and Spinal Cord Trauma
  • 180. 181 of 17 Shock 3 Discussion Questions: 4. How should you initially manage this patient? 5. What would you do if the patient was not breathing? Case Details M 28-year-old male fell 5 meters (16 feet) from scaffolding, wearing hard hat; bystander reports patient landed head- first, with neck hyperextended None reported Skin warm; breathing shallow; not moving arms or legs; Vital signs: BP 80/62; HR 58; RR 28; GCS 15 None Reported I S T 7 Spine and Spinal Cord Trauma
  • 181. 182 of 17 Shock 3 Discussion Questions: 6. Which signs and symptoms are concerning for a spinal injury in this patient? 7. What other signs and symptoms not previously mentioned may be associated with the presence of a spinal cord injury? Case Details M 28-year-old male fell 5 meters (16 feet) from scaffolding, wearing hard hat; bystander reports patient landed head- first, with neck hyperextended None reported Skin warm; breathing shallow; not moving arms or legs; Vital signs: BP 80/62; HR 58; RR 28; GCS 15 None Reported I S T 7 Spine and Spinal Cord Trauma
  • 182. 183 of 17 Shock 3 Case Scenario Progression Peripheral neurologic exam: • Patient unable to move or feel legs • Patient can move fingers and wrists bilaterally • Weak L triceps extension • Patient unable to move R elbow • Patient able to feel fingers and thumbs bilaterally • Patient unable to feel inner arms above the elbows 7 Spine and Spinal Cord Trauma
  • 183. 184 of 17 Shock 3 Discussion Questions: 1. At what level is the suspected spinal lesion? 2. What resources are available to assist with determining the level of spinal injury? 3. Why is the exam different between the right and left upper extremities? • Patient unable to move or feel legs • Patient can move fingers and wrists bilaterally • Weak L triceps extension • Patient unable to move R elbow • Patient able to feel fingers and thumbs bilaterally • Patient unable to feel inner arms above the elbows Case Details 7 Spine and Spinal Cord Trauma
  • 184. 185 of 17 Shock 3 Discussion Questions: 4. What are important aspects of documenting the neurologic examination? 5. Why is it important to accurately and thoroughly document the neurologic examination findings? 7 Spine and Spinal Cord Trauma
  • 185. 186 of 17 Shock 3 Case Scenario Progression • Patient is spinal motion restricted • 1 L of crystalloid fluid is given • Vital signs: BP 100/64; HR 62; RR 28; GCS 15 7 Spine and Spinal Cord Trauma
  • 186. 187 of 17 Shock 3 Discussion Questions: 1. What imaging would you request for this patient? 2. What tools are available to assist with decisions regarding spine imaging? • Patient is spinal motion restricted • 1 L of crystalloid fluid is given • Vital signs: BP 100/64; HR 62; RR 28; GCS 15 Case Details 7 Spine and Spinal Cord Trauma
  • 187. 188 of 17 Shock 3 Case Scenario Progression • Negative chest x-ray, pelvic x-ray, and FAST exam • CT scan unavailable. • Lateral c-spine x-ray: C6 fracture with anterior displacement. • Patient placed in c-collar with mobility restrictions • Vital signs: BP 80/62; HR 58; RR 28; GCS 15 7 Spine and Spinal Cord Trauma
  • 188. 189 of 17 Shock 3 Discussion Question: How do you manage the patient’s ABCs at this point? • Negative chest x-ray, pelvic x-ray, and FAST exam • CT scan unavailable. • Lateral c-spine x-ray: C6 fracture with anterior displacement. • Patient placed in c-collar with mobility restrictions • Vital signs: BP 80/62; HR 58; RR 28; GCS 15 Case Details 7 Spine and Spinal Cord Trauma
  • 189. 190 of 17 Shock 3 Case Scenario Progression • Following additional 1 L of IV crystalloid, BP remains 80/50, HR 45 • RR increased to 30 • Patient complains of shortness of breath • ABGs: PCO2 50 mm Hg 7 Spine and Spinal Cord Trauma
  • 190. 191 of 17 Shock 3 Discussion Questions: 1. What interventions are indicated at this facility? 2. Does this patient require transfer? What is the rationale for this decision? • Following additional 1 L of IV crystalloid, BP remains 80/50, HR 45 • RR increased to 30 • Patient complains of shortness of breath • ABGs: PCO2 50 mm Hg Case Details 7 Spine and Spinal Cord Trauma
  • 191. 192 of 17 Shock 3 Case Scenario Conclusion • Patient intubated • Systolic BP improves to 110/65 with vasopressor support • O2 sat 98% on 30% oxygen • Repeat FAST negative • Transferred to definitive care, where C6 and T6 fractures identified • Admitted to ICU • Surgery for C6 fracture, nonoperative management of T6 fracture • Transferred to a spinal cord rehabilitation center 7 Spine and Spinal Cord Trauma
  • 192. 193 of 17 Shock 3 Any Questions? 7 Spine and Spinal Cord Trauma
  • 193. 194 of 17 Shock 3 Review Objectives By the end of this interactive discussion, you will be able to: 1. Apply the ABC principles of ATLS when assessing a patient for spine injury. 2. Identify a common mechanism and type of spinal injury. 3. Describe the typical signs and symptoms of a patient with a spinal cord injury. 4. Describe the technique and importance of documentation of a potential spinal injury. 5. Describe the appropriate initial treatment of patients with spinal injuries. 6. Determine the appropriate disposition of patients with spine trauma. 7 Spine and Spinal Cord Trauma
  • 194. 195 of 17 Shock 3 Key Learning Points 1. Attend to the life-threatening injuries identified in the primary survey while minimizing movement of the spine. 2. Assume possible spinal injury until clinical and/or radiographic evaluation can be completed (decision tools such as Canadian C-Spine Rules or NEXUS may be used). 3. Be as specific and accurate as possible when describing and documenting the level of neurologic injury (ASIA tool is extremely useful). 4. High spinal cord injuries may be associated with respiratory failure and/or neurogenic shock, which must be addressed prior to transfer. 5. Consider obtaining early consultation with a spine surgeon when a spinal injury is suspected and/or detected. 7 Spine and Spinal Cord Trauma
  • 196. 197 of 17 Shock 3 Case Scenario #1 A 65-year-old male is transported to the emergency department by EMS after a motor vehicle collision while driving home at 50 km/hr (30 mph) from a tavern. He is currently being treated for a recent deep vein thrombosis. He has a left scalp contusion. He has a strong odor of alcohol on his breath. He is thought to have suffered a concussion and to have alcohol intoxication. His airway is clear, he is breathing spontaneously without difficulty, and he has no hemodynamic abnormalities. His eyes are open. He appears confused and pushes away the examiner’s hands when assessed for response to fingertip pressure. He has a cervical collar in place, and motion is restricted on a long spine board. M I S T GCS ?
  • 197. 198 of 17 Shock 3 Case Scenario #1 A 65-year-old male is transported to the emergency department by EMS after a motor vehicle collision while driving home at 50 km/hr (30 mph) from a tavern. He is currently being treated for a recent deep vein thrombosis. He has a left scalp contusion. He has a strong odor of alcohol on his breath. He is thought to have suffered a concussion and to have alcohol intoxication. His airway is clear, he is breathing spontaneously without difficulty, and he has no hemodynamic abnormalities. His eyes are open. He appears confused and pushes away the examiner’s hands when assessed for response to fingertip pressure. He has a cervical collar in place, and motion is restricted on a long spine board. M I S T GCS ? E : 4 V : 4 (?) M : 4 GCS 12 (?)
  • 198. 199 of 17 Shock 3 Case Scenario #3 A 75-year-old male is brought to a rural emergency department by his family after a ground-level fall after slipping on a water puddle. According to his family, he had an approximate 30-second loss of consciousness. The patient does not remember the injury event and only complains of a headache. His past medical history is only significant for hypertension. Other than a small left frontoparietal soft tissue contusion, there are no other gross injuries. His vital signs are: BP 140/85, HR 70, and RR 16. Other than a small left frontoparietal soft tissue contusion, there are no other gross injuries. He is awake, opens his eyes spontaneously, follows commands, communicates appropriately, and does not have any focal neurological deficits. Triage personnel put patient in cervical collar. M I S T GCS ?
  • 199. 200 of 17 Shock 3 Case Scenario #3 A 75-year-old male is brought to a rural emergency department by his family after a ground-level fall after slipping on a water puddle. According to his family, he had an approximate 30-second loss of consciousness. The patient does not remember the injury event and only complains of a headache. His past medical history is only significant for hypertension. Other than a small left frontoparietal soft tissue contusion, there are no other gross injuries. His vital signs are: BP 140/85, HR 70, and RR 16. Other than a small left frontoparietal soft tissue contusion, there are no other gross injuries. He is awake, opens his eyes spontaneously, follows commands, communicates appropriately, and does not have any focal neurological deficits. Triage personnel put patient in cervical collar. I S T GCS ? E : 4 V : 5 M : 6 GCS 15 M
  • 200. 201 of 17 Shock 3 Disability Video 1 Helmet Removal
  • 201. 202 of 17 Shock 3 Disability Video 2 Logrolling Technique
  • 202. 203 of 17 Shock 3 CT of head : epidural hematoma Epidural Hematoma
  • 203. 204 of 17 Shock 3 CT of head : subdural hematoma Subdural Hematoma
  • 204. 205 of 17 Shock 3 CT of head : subarachnoid hemorrhage and hemorrhage on the left frontoparietal area Subarachnoid Hemorrhage
  • 205. 206 of 17 Shock 3 CT of head : intracerebral hematoma Intracerebral Hematoma
  • 207. 208 of 17 Shock 3 Injuries to the musculoskeletal system are common in trauma patients. The delayed recognition and treatment of these injuries can result in life-threatening hemorrhage or limb loss. 8 Musculoskeletal Trauma
  • 208. 209 of 17 Shock 3 Objectives By the end of this interactive discussion, you will be able to: 1. Explain the significance of musculoskeletal injuries in patients with multiple injuries. 2. Outline the priorities of the primary survey, resuscitation and secondary survey of patients with extremity injuries. 3. Identify the adjuncts needed in the immediate treatment of life-threatening extremity hemorrhage. 4. Explain the principles of the initial management of limb-threatening musculoskeletal injuries. 8 Musculoskeletal Trauma
  • 209. 210 of 17 Shock 3 Case Scenario 38-year-old female (102 kg), restrained driver, high-speed, head-on collision with large truck Prolonged extrication, marked deformity R thigh, open fracture R lower leg. Awake and alert, in severe pain; Vital signs: HR 120; BP 90/50; RR 22; GCS 15 Two large-bore IVs in upper extremities, resuscitation with isotonic fluids M I S T 8 Musculoskeletal Trauma
  • 210. 211 of 17 Shock 3 Case Scenario Progression • Airway and breathing are OK • Lower limb actively bleeding, dressing applied • Vital signs after fluids: HR 130; BP 80/40; RR 24; GCS 14 • Chest x-ray, pelvic x-ray, and FAST exam negative 8 Musculoskeletal Trauma
  • 211. 212 of 17 Shock 3 Discussion Questions: 1. What are the priorities for this patient during the primary survey and resuscitation? 2. How will you assess the injured extremities at this point? Case Details • Airway and breathing are OK • Lower limb bleeding, dressing applied • Vital signs after fluids: HR 130; BP 80/40; RR 24; GCS 14 • Chest x-ray, pelvic x-ray, and FAST exam negative 8 Musculoskeletal Trauma
  • 212. 213 of 17 Shock 3 Discussion Questions: 3. How much blood loss would you expect from this patient’s extremity injuries, and what is the best way to control it? 4. How should femur and tibial shaft fractures be stabilized? Case Details • Airway and breathing are OK • Lower limb bleeding, dressing applied • Vital signs after fluids: HR 130; BP 80/40; RR 24; GCS 14 • Chest x-ray, pelvic x-ray, and FAST exam negative 8 Musculoskeletal Trauma
  • 213. 214 of 17 Shock 3 Case Scenario Progression • Right dorsalis pedis and posterior tibial pulses absent • Pulses remain absent after splinting. • Blood soaking through dressings, R lower leg • Dressing taken down, pulsatile bleeding noted • Direct pressure applied to wound, followed by gauze packing • Dressing quickly becomes saturated with blood again 8 Musculoskeletal Trauma
  • 214. 215 of 17 Shock 3 Discussion Question: What is the next step in the management of this patient’s uncontrolled extremity hemorrhage? Case Details • Right dorsalis pedis and posterior tibial pulses absent • Pulses remain absent after splinting. • Blood soaking through dressings, R lower leg • Dressing taken down, pulsatile bleeding noted • Direct pressure applied to wound, followed by gauze packing • Dressing quickly becomes saturated with blood again 8 Musculoskeletal Trauma
  • 215. 216 of 17 Shock 3 Case Scenario Progression • Tourniquet applied and time of placement documented • Bleeding controlled • Distal pulses not palpable 8 Musculoskeletal Trauma
  • 216. 217 of 17 Shock 3 Discussion Question: How would you manage the patient now? • Tourniquet applied and time of placement documented • Bleeding controlled • Distal pulses not palpable Case Details 8 Musculoskeletal Trauma
  • 217. 218 of 17 Shock 3 Case Scenario Progression • Patient’s hemodynamics improve with IV fluids • Secondary survey begins • Awaiting transfer due to no surgical capabilities at this facility 8 Musculoskeletal Trauma
  • 218. 219 of 17 Shock 3 Discussion Question: 1. How would you identify any limb- threatening injuries or other extremity injuries during the secondary survey? 2. What x-rays should be ordered for this patient prior to transfer? • Patient’s hemodynamics improve • Secondary survey begins • Awaiting transfer due to no surgical capabilities at this facility Case Details 8 Musculoskeletal Trauma
  • 219. 220 of 17 Shock 3 Case Scenario Progression • X-rays of the patient’s extremities are obtained. • Radiographs do not cause a delay in transfer. 8 Musculoskeletal Trauma
  • 220. 221 of 17 Shock 3 Case Scenario Progression • Vital signs: HR 105; BP 110/70; RR 24; GCS 15. • Bleeding is controlled with tourniquet. • Urgent transfer already initiated. • No known drug allergies. 8 Musculoskeletal Trauma
  • 221. 222 of 17 Shock 3 Discussion Question: 1. What else should be done prior to transfer 2. What antibiotics would you give and at what dose? 3. How can you decrease the patient’s discomfort and pain? Case Details • Vital signs: HR 105; BP 110/70; RR 24; GCS 15. • Bleeding is controlled with tourniquet. • Your facility does not have the resources to manage the patient’s injuries, so urgent transfer initiated. 8 Musculoskeletal Trauma
  • 222. 223 of 17 Shock 3 Case Scenario Progression • The patient is transferred to the nearest trauma center, where a trauma surgeon and an orthopedic surgeon are available to take the patient to the operating room immediately • Vital signs: HR 105; BP 110/70; RR 24; GCS 15 • Bleeding is controlled with tourniquet • Patient received antibiotics prior to transfer 8 Musculoskeletal Trauma
  • 223. 224 of 17 Shock 3 Discussion Question: 1. What measures need to be taken at the receiving hospital? 2. How can you make an early diagnosis of compartment syndrome? 3. What is the treatment for compartment syndrome? Case Details • Patient is transferred to the nearest trauma center, where a trauma surgeon and an orthopedic surgeon are available to take the patient to the operating room immediately. • Vital signs: HR 105; BP 110/70; RR 24; GCS 15. • Bleeding is controlled with tourniquet. • Patient received antibiotics prior to transfer. 8 Musculoskeletal Trauma
  • 224. 225 of 17 Shock 3 Case Scenario Conclusion Patient taken to the operating room for urgent evaluation of vascular injury, wound washout and external fixation of fracture. 8 Musculoskeletal Trauma
  • 225. 226 of 17 Shock 3 Any Questions? 8 Musculoskeletal Trauma
  • 226. 227 of 17 Shock 3 Review Objectives By the end of this interactive discussion, you will be able to: 1. Explain the significance of musculoskeletal injuries in patients with multiple injuries. 2. Outline the priorities of the primary survey and resuscitation of patients with extremity injuries. 3. Identify the adjuncts needed in the immediate treatment of life-threatening extremity hemorrhage. 4. Describe key elements of the secondary survey of patients with musculoskeletal trauma. 5. Explain the principles of the initial management of limb-threatening musculoskeletal injuries. 8 Musculoskeletal Trauma
  • 227. 228 of 17 Shock 3 Key Learning Points • Hemorrhage from long bone fractures can be significant • Early splinting helps to control blood loss, reduce pain, and prevent further neurovascular compromise and soft tissue injury • Early weight-based dosing of antibiotics for patients with open fractures • Compartment syndrome is a clinical diagnosis, and the treatment is fasciotomy. 8 Musculoskeletal Trauma
  • 229. Normal Chest X-ray - Patient identity - Density - Centration
  • 230. 231 of 17 Shock 3 Chest X-ray Massive left hematothorax
  • 231. 232 of 17 Shock 3 Chest X-ray - Right pneumothorax (collaps of right lung) - Left pulmonary contusion
  • 232. 233 of 17 Shock 3 Chest X-ray - Left pneumothorax (chest tube inserted) - Multiple rib fractures - Left subcutaneous air
  • 233. 234 of 17 Shock 3 CXR Chest X-ray - Multiple rib fractures (flail chest) - Left subcutaneous air - Fractures of : -clavicle -humeral neck -scapula
  • 234. Chest X-ray - Abnormal gas left chest (herniation of intraabdominal organ, bowel) - Diaphragm not visualized (rupture of left diaphragma)
  • 235. 236 of 17 Shock 3 Chest X-ray Widened mediastinum on CXR
  • 236. 237 of 17 Shock 3 Chest CT-scan - Widened mediastinum - Periaortic hematoma (aortic injury)
  • 237. 238 of 17 Shock 3 Normal Pelvic X-ray
  • 238. 239 of 17 Shock 3 Pelvic X-ray - Extensive widening of the sacroiliac joints - Pelvic diastasis
  • 239. 240 of 17 Shock 3 Pelvic X-ray Bilateral inferior and superior pelvic rami fractures
  • 240. 241 of 17 Shock 3 Pelvic Binder before binder after binder Foto : pelvic diasthesis
  • 241. 242 of 17 Shock 3 Pelvic X-ray Unconscious and hypotensive Foto : vertical shear fracture 1. vertical translation of the pelvis and widened pubis 2. extensive widening of the sacroiliac joints 3. inferior and superior pelvic rami fractures 4. left acetabular fractures
  • 242. Urethrogram Foto : No extravasation of contrast material
  • 243. 244 of 17 Shock 3 Urethro-cyctogram Clinical : - Hypotension - obvious unstable pelvis - gross hematuria - perineal ecchymosis Foto : No extravasation of contrast material
  • 244. 245 of 17 Shock 3 Urethro-cyctogram Clinical : Gross hematuria Foto : Extravasation of contrast material outsite bladder, but confined to pelvis (retroperitoneal bladder rupture)
  • 245. 246 of 17 Shock 3 Urethro-cyctogram Clinical : - lower abdominal pain - gross hematuria Foto : - diffuse contrast extravasation - not confined to the pelvis (intraperitoneal bladder rupture)
  • 246. 247 of 17 Shock 3 A Alignment and adequacy A. Anterior vertebral line B. Anterior spinal line C. Posterior spinal line D. Spinous process Cervical Spine Anatomy
  • 247. 248 of 17 Shock 3 B Bone C1 Atlas C2 Odontoid C3-C7 C2 C1 C7 Cervical Spine Anatomy
  • 248. 249 of 17 Shock 3 C Cartilage D Dens E Extra axial soft tissues F Facets Cervical Spine Anatomy
  • 249. 250 of 17 Shock 3 Pediatric Cervical Spine normal subluxation of C2-C3
  • 251. 252 of 17 Shock 3 Cervical Spine odontoid fracture, comminution and depression at the C1-C2 interfacet joint
  • 252. Open Mouth and Lateral View C-1 burst (Jefferson fracture)
  • 253. 254 of 17 Shock 3 Cervical Spine complete subluxation of C7-T1 subluxation of C6-C7
  • 254. 255 of 17 Shock 3 Thoraco-lumbal Spine fracture dislocation of T3-T4 fracture of L2 fracture of L3
  • 256. 257 of 17 Shock 3 The most significant difference between burns and other injuries is that the consequences of burn injury are directly linked to the extent of the inflammatory response to the injury. 9 Thermal Injuries
  • 257. 258 of 17 Shock 3 Objectives By the end of this interactive discussion, you will be able to: 1. Discuss the potential risks to the airway of patients with burn injuries. 2. Discuss resuscitation strategies for patients with burns. 3. Estimate the extent of a simulated patient’s burn injury. 4. Describe the appropriate management of burn injuries, including circumferential burns. 5. Discuss the proper handover method for patients with burns. 6. Describe management of patients with hypothermia, including rewarming risks. 7. Describe the tissue effects of cold injury. 8. Describe the initial treatment of patients with tissue injury from cold exposure. 9 Thermal Injuries
  • 258. 259 of 17 Shock 3 Case Scenario 29-year-old male jumps from first story of a burning house; clothes on fire; bystanders extinguish fire and summon EMS Conscious, agitated, voice normal, complaining of abdominal and leg pain; head and upper body extensively burned None reported None reported M I S T 9 Thermal Injuries
  • 259. 260 of 17 Shock 3 Discussion Questions: 1. What are your immediate priorities? 2. Is this patient’s airway at risk? Why, or why not? 3. What are some physical examination findings that suggest the airway is injured? Case Details M 29-year-old male jumps from first story of a burning house; clothes on fire; bystanders extinguish fire and summon EMS Conscious, agitated, voice normal, complaining of abdominal and leg pain; head and upper body extensively burned None reported None reported I S T 9 Thermal Injuries
  • 260. 261 of 17 Shock 3 Discussion Questions: 4. How can you decrease injury progression from the burn? 5. What are ways to control this patient’s pain? Case Details M 29-year-old male jumps from first story of a burning house; clothes on fire; bystanders extinguish fire and summon EMS Conscious, agitated, voice normal, complaining of abdominal and leg pain; head and upper body extensively burned None reported None reported I S T 9 Thermal Injuries
  • 261. 262 of 17 Shock 3 Discussion Questions: 6. What other potential injuries do you need to consider in this patient? 7. What other thermal injury consequences do you need to consider? Case Details M 29-year-old male jumps from first story of a burning house; clothes on fire; bystanders extinguish fire and summon EMS Conscious, agitated, voice normal, complaining of abdominal and leg pain; head and upper body extensively burned None reported None reported I S T 9 Thermal Injuries
  • 262. 263 of 17 Shock 3 Case Scenario Progression • Patient is intubated • IV access obtained in the antecubital fossa through burned skin • Patient exposed fully: face, anterior torso, and bilateral lower extremities nearly completely burned circumferentially • Foley catheter placed, minimal dark urine 9 Thermal Injuries
  • 263. 264 of 17 Shock 3 Discussion Question: 1. What is estimated size of the burn in this patient? 9 Thermal Injuries
  • 264. 265 of 17 Shock 3 Discussion Questions: 2. What is the significance of the dark urine? Case Details • Patient is intubated • IV access obtained in the antecubital fossa through burned skin • Patient exposed fully: face, anterior torso, and bilateral lower extremities nearly completely burned circumferentially • Foley catheter placed, minimal dark urine 9 Thermal Injuries
  • 265. 266 of 17 Shock 3 Case Scenario Progression • Initial ABCDEs addressed • Estimated TBSA burned calculated • Second- and third-degree burn wounds throughout 9 Thermal Injuries
  • 266. 267 of 17 Shock 3 Discussion Question: How will you manage this patient’s burns to prevent further morbidity and mortality? • Initial ABCDEs addressed • Estimated TBSA burned calculated • Second- and third- degree burn wounds throughout Case Details 9 Thermal Injuries
  • 267. 268 of 17 Shock 3 Case Scenario Progression • Fluid resuscitation begun • Urine output < 30 mL/hr • Fluid rate increased • Chest and extremity x-rays ordered 9 Thermal Injuries
  • 268. 269 of 17 Shock 3 Discussion Question: How do resuscitation strategies differ in burn resuscitation from other types of trauma resuscitation? • Fluid resuscitation begun • Urine output < 30 mL/hr • Fluid rate increased • Chest and extremity x- rays ordered Case Details 9 Thermal Injuries 9 Thermal Injuries
  • 269. 270 of 17 Shock 3 Case Scenario Progression • Secondary survey: calves of both legs firm and edematous 9 Thermal Injuries
  • 270. 271 of 17 Shock 3 Discussion Question: What is the significance of finding swelling and firm compartments on the secondary survey? • Calves of both legs firm and edematous Case Details 9 Thermal Injuries
  • 271. 272 of 17 Shock 3 Case Scenario Conclusion • Transfer arrangements made • Decision to perform escharotomies prior to transfer 9 Thermal Injuries
  • 272. 273 of 17 Shock 3 Discussion Question: As the team prepares for transfer and to carry out the hand-off to the receiving facility, what are some important elements that should be communicated, and where should they be documented? • Transfer arrangements made • Decision to perform escharotomies prior to transfer Case Details 9 Thermal Injuries
  • 273. 274 of 17 Shock 3 Case Scenario #2 35-year-old female brought to hospital after being lost for two days, snowmobiling in -30° C (-22° F) weather None reported Core body temperature 30° C (86° F) None reported M I S T 9 Thermal Injuries
  • 274. 275 of 17 Shock 3 Discussion Questions: 1. How would you initially treat this patient? 2. How and why would you monitor this patient during the rewarming? Case Details M 35-year-old female brought to hospital after being lost for two days, snowmobiling in -30° C weather None reported Core body temperature 30° C None reported I S T 9 Thermal Injuries
  • 275. 276 of 17 Shock 3 Any Questions? 9 Thermal Injuries
  • 276. 277 of 17 Shock 3 Objectives By the end of this interactive discussion, you will be able to: 1. Discuss the potential risks to the airway of patients with burn injuries. 2. Discuss resuscitation strategies for patients with burns. 3. Estimate the extent of a simulated patient’s burn injury. 4. Describe the appropriate management of burn injuries, including circumferential burns. 5. Discuss the proper handover method for patients with burns. 6. Describe management of patients with hypothermia, including rewarming risks. 7. Describe the tissue effects of cold injury. 8. Describe the initial treatment of patients with tissue injury from cold exposure. 9 Thermal Injuries
  • 277. 278 of 17 Shock 3 Key Learning Points 1. The most significant difference between burns and other injuries is that the consequences of burn injury are directly linked to the extent of the inflammatory response to the injury. This drives the rate and amount of edema formation. 2. The airway can become obstructed not only from direct injury (e.g., inhalation injury), but also from the massive edema resulting from the burn injury. Edema is typically not present immediately, and signs of obstruction may initially be subtle until the patient is in crisis. 3. In contrast to resuscitation for other types of trauma in which fluid deficit is typically secondary to hemorrhagic losses, burn resuscitation is required to replace the ongoing losses from capillary leak due to inflammation. 9 Thermal Injuries
  • 278. 279 of 17 Shock 3 Key Learning Points 4. A fresh burn is a clean area that must be protected from contamination. 5. Ensure that there are flow sheets documenting the patient history, injury, IV fluids given, and urinary output. The flow sheet should be sent with the patient on transfer. 6. Although rapid rewarming is essential for management of frostbite and hypothermia, reperfusion can cause physiologic changes that need to be managed. 9 Thermal Injuries
  • 279. 10 Pediatric Trauma Tenth Edition
  • 280. 281 of 17 Shock 3 Injury remains the most common cause of death and disability in childhood. Injury morbidity and mortality surpass all major diseases in children and young adults, making trauma the most serious public health and health care problem in this population. 10 Pediatric Trauma
  • 281. 282 of 17 Shock 3 Objectives By the end of this interactive discussion, you will be able to: 1. Identify the initial priorities of trauma assessment and management for children. 2. Describe the most appropriate interventions for managing difficult airways in pediatric trauma patients. 3. Recognize the most common causes of cardiac arrest in children. 4. Identify methods for obtaining venous access in children. 5. Discuss how to determine drug and fluid dosages in children. 6. Evaluate for nonaccidental trauma in a pediatric trauma case. 10 Pediatric Trauma
  • 282. 283 of 17 Shock 3 Case Scenario 3-year-old boy falls 10 meters (32 feet) out of an apartment window onto pavement Patient does not open eyes, moans incomprehensibly, extends abnormally when stimulated Unresponsive on arrival to ED at a small rural hospital, pupils unequal, blood coming from R ear, breathing rapidly, pale, mottled extremities Vital signs: BP 74/57; HR 156; RR 49 None reported M I S T 10 Pediatric Trauma
  • 283. 284 of 17 Shock 3 Discussion Question: What are the priorities in evaluating a small child with multisystem trauma? Case Details M 3-year-old boy falls 10 meters (32 ft) out of an apartment window onto pavement Patient does not open eyes, moans incomprehensibly, extends abnormally when stimulated Unresponsive on arrival to ED at a small rural hospital, pupils unequal, blood coming from R ear, breathing rapidly, pale, mottled extremities Vital signs: BP 74/57; HR 156; RR 49 None reported I S T 10 Pediatric Trauma
  • 284. 285 of 17 Shock 3 Case Scenario Progression • Vital signs: BP 74/57; HR 156; RR 49; O2 sat 85% • Breath sounds symmetrical 10 Pediatric Trauma
  • 285. 286 of 17 Shock 3 Discussion Question: 1. What is the most common cause of cardiac arrest in children? 10 Pediatric Trauma
  • 286. 287 of 17 Shock 3 Discussion Questions: 2. What steps and maneuvers would you use to manage this patient’s airway? 3. Based on the information given, is this child in shock? If so, what type of shock is exhibited? Case Details • Vital signs: BP 74/57; HR 156; RR 49; O2 sat 85% • Breath sounds symmetrical 10 Pediatric Trauma
  • 287. 288 of 17 Shock 3 Case Scenario Progression • Two failed attempts at intubation • Attempt to position laryngeal mask unsuccessful 10 Pediatric Trauma
  • 288. 289 of 17 Shock 3 Discussion Question: What would your next step be in securing an airway for this patient? 10 Pediatric Trauma
  • 289. 290 of 17 Shock 3 Case Scenario Progression • Needle cricothryroidotomy performed, airway secured • Vital signs: BP 74/57; HR 156; RR 49 • Several failed attempts at peripheral IV access in both antecubital fossae 10 Pediatric Trauma
  • 290. 291 of 17 Shock 3 Discussion Question: 1. How would you secure vascular access to give fluids? 2. How would you manage this patient’s hypotension and determine the appropriate fluid regime? • Needle cricothryroidotomy performed, airway secured • Vital signs: BP 74/57; HR 156; RR 49 • Several failed attempts at peripheral IV access in both antecubital fossae Case Details 10 Pediatric Trauma
  • 291. 292 of 17 Shock 3 Case Scenario Progression • Isotonic crystalloid and O-negative blood given with good response • Vital signs: HR 110; BP 90/60 • Chest x-ray: pulmonary contusions 10 Pediatric Trauma
  • 292. 293 of 17 Shock 3 Discussion Question: Why do children commonly develop pulmonary contusions following trauma, even in the absence of rib fractures? 10 Pediatric Trauma
  • 293. 294 of 17 Shock 3 Case Scenario Progression • Pelvic x-ray: normal • Inconsistencies noted in family’s explanation of fall • Skeletal survey: healed transphyseal distal L humerus fracture; healing classic L femur metaphyseal lesion, with periostitis extending into the femoral diaphysis 10 Pediatric Trauma
  • 294. 295 of 17 Shock 3 Discussion Questions: 1. Do the x-ray findings raise the suspicion of child maltreatment? If so, why? 2. What are the appropriate steps to take if child maltreatment is suspected? • Pelvic x-ray: normal • Inconsistencies noted in family’s explanation of fall • Skeletal survey: healed transphyseal distal L humerus fracture; healing classic L femur metaphyseal lesion, with periostitis extending into the femoral diaphysis Case Details 10 Pediatric Trauma
  • 295. 296 of 17 Shock 3 Case Scenario Conclusion • Transfer to nearest pediatric trauma center after intubation and normalization of hemodynamics • CT of head and abdomen at receiving facility: intracerebral contusions, moderate splenic contusion, and subscapular hematoma • Managed in ICU until gas exchange normalized, extubated • Splenic injury does not require surgery • Case reported to the appropriate social and legal services • Outpatient brain injury follow up for 6 months 10 Pediatric Trauma
  • 296. 297 of 17 Shock 3 Any Questions? 10 Pediatric Trauma
  • 297. 298 of 17 Shock 3 Review Objectives By the end of this interactive discussion, you will be able to: 1. Identify the initial priorities of trauma assessment and management for children. 2. Describe the most appropriate interventions for managing difficult airways in pediatric trauma patients. 3. Recognize the most common causes of cardiac arrest in children. 4. Identify methods for obtaining venous access in children. 5. Discuss how to determine drug and fluid dosages in children. 6. Evaluate for nonaccidental trauma in a pediatric trauma case. 10 Pediatric Trauma
  • 298. 299 of 17 Shock 3 Key Learning Points 1. The initial priorities of trauma assessment and management are the same for children and adults. 2. Surgical cricothyroidotomy is generally considered to be unsafe in small children (<12) due to the small size of the cricothyroid membrane and proximity to vocal cords. Needle cricothyroidotomy is preferred as a temporizing solution until other preparations are made. 3. Hypoxia and respiratory compromise are the most common causes of cardiac arrest in children. 4. Emergent venous access in children can be difficult. If unable to obtain peripheral access, intraosseous access should be obtained immediately. 10 Pediatric Trauma
  • 299. 300 of 17 Shock 3 Key Learning Points 5. Determination of weight is essential to pediatric trauma care in order to dose drugs and guide fluid and blood resuscitation. This can be accomplished by history, length-based resuscitation tape, or specialty stretchers with integrated scales. 6. Blunt solid organ injury in pediatric patients is usually managed non-operatively by a surgeon unless the patient is hemodynamically unstable or there are other indications for surgery. 7. Non-accidental trauma is a significant source of injury in children and has a higher mortality rate than corresponding accidental injuries. Specific injury patterns exist that should heighten concerns. Clinicians must have a high index of suspicion and report these cases appropriately. 10 Pediatric Trauma
  • 300. 10 Pediatric Trauma Tenth Edition
  • 301. 302 of 17 Shock 3 When managing geriatric patients with trauma, the effects of aging on physiological function and the impact of preexisting conditions and medications cannot be overemphasized. 11 Geriatric Trauma
  • 302. 303 of 17 Shock 3 Objectives By the end of this interactive discussion, you will be able to: 1. Describe common mechanisms of injury seen in older adults. 2. Apply the ATLS principles to the management of an elderly trauma patient. 3. Understand the physiologic changes that occur with aging and how they affect the geriatric patient’s injury and response to trauma. 4. Understand the common signs and causes of elder maltreatment. 11 Geriatric Trauma
  • 303. 304 of 17 Shock 3 Case Scenario 82-year-old female fell down 6 stairs; husband called ambulance, transported to your rural ED None reported Vital signs: RR 22; HR 64; BP 160/80; GCS 13 None reported M I S T 11 Geriatric Trauma
  • 304. 305 of 17 Shock 3 Discussion Questions: 1. What injuries would you suspect in this elderly patient? 2. What unique elements of the AMPLE history should you be alert to? 3. What are the key aspects of the initial assessment and management of this patient? Case Details M 82-year-old female fell down 6 stairs; husband called ambulance, transported to your rural ED None reported Vital signs: RR 22; HR 64; BP 160/80; GCS 13 None reported I S T 11 Geriatric Trauma
  • 305. 306 of 17 Shock 3 Case Scenario Progression • Examination • Vital signs unchanged on ED arrival • Patient confused, localizes to pressure • Multiple bruises • Less movement upper compared with lower extremities • AMPLE: taking a direct thrombin inhibitor, beta blocker for hypertension • Investigations: • Chest x-ray: multiple rib fractures • Head CT: subdural hematoma with shift, few small intracerebral contusions • Pelvic x-ray: pubic rami fractures 11 Geriatric Trauma
  • 306. 307 of 17 Shock 3 Discussion Questions: 1. What challenges and pitfalls might you encounter if you need to intubate the patient? 2. With respect to her rib fractures, why is this patient at higher risk for mortality than a younger individual? Case Details • Vital signs unchanged on ED arrival • Patient confused, localizes to pressure • Multiple bruises • Less movement upper compared with lower extremities • AMPLE: taking a direct thrombin inhibitor, beta blocker for hypertension • Chest x-ray: multiple rib fractures • Head CT: subdural hematoma with shift, few small intracerebral contusions • Pelvic x-ray: pubic rami fractures 11 Geriatric Trauma
  • 307. 308 of 17 Shock 3 Case Scenario Progression • Transfer not possible due to severe weather • Admitted to ICU for monitoring • Rib fracture pain controlled with narcotics • Respiratory status and level of consciousness continue to decline • Repeat head CT: progression of subdural hematoma • Neurosurgeon by telemedicine recommends craniotomy for evacuation of the hematoma when transfer is available 11 Geriatric Trauma
  • 308. 309 of 17 Shock 3 Discussion Questions: 1. What are the potential causes of declining mental status in this patient? What are the most likely? 2. What is the impact of a prolonged ICU admission and hospital stay on this elderly patient’s likelihood of returning to independent living? Case Details • Transfer not possible due to severe weather • Admitted to ICU for monitoring • Rib fracture pain controlled with narcotics • Respiratory status and level of consciousness continue to decline • Repeat head CT: progression of subdural hematoma • Neurosurgeon by telemedicine recommends craniotomy for evacuation of the hematoma when transfer is available 11 Geriatric Trauma
  • 309. 310 of 17 Shock 3 Discussion Questions: 3. Does the patient’s injury pattern raise suspicion about elder maltreatment? 4. What type of injuries would raise your suspicion about elder maltreatment? Case Details M 82-year-old female fell down 6 stairs; husband called ambulance, transported to your rural ED None reported Vital signs: RR 22; HR 64; BP 160/80; GCS 13 None reported I S T 11 Geriatric Trauma
  • 310. 311 of 17 Shock 3 Case Scenario Conclusion • Husband states that the patient would not want intubation nor life- sustaining interventions if there is no chance of living independently again. • He provides advance directive and living will documenting her wishes and his status as health care power of attorney. 11 Geriatric Trauma
  • 311. 312 of 17 Shock 3 Any Questions? 11 Geriatric Trauma